Crime MCQS Flashcards

1
Q

Which of the following is not a key part of the criminal justice process as discussed in this element?

A) Arrest

B) Appeal

C) Trial

D) The prevention of crime

E) Plea

A

D) The prevention of crime

(Correct. The key elements of the criminal justice process discussed in this element were: arrest/ requisition, plea, trial, sentencing and appeals. The prevention of crime is more of a matter for criminology studies but does become a factor when sentencing someone found guilty of a crime)

How well did you know this?
1
Not at all
2
3
4
5
Perfectly
2
Q

All of the following bodies can bring prosecutions. However, which of the following is the main body which brings prosecutions in England and Wales?

A) Health and safety executive

B) Transport for London

C) Crown Prosecution Service

D) Private individuals

A

C) Crown Prosecution Service

(Correct. Other bodies such as Transport for London, railway operators and the Health and Safety Executive prosecute their own matters. Private individuals can bring prosecutions too but the CPS has the right to intervene and take over such cases)

How well did you know this?
1
Not at all
2
3
4
5
Perfectly
3
Q

What classification of offence is murder?

A) Indictable-only

B) Summary only

C) Either-way

A

A) Indictable-only

(This is the correct answer. Indictable-only offences like murder are the most serious offences)

How well did you know this?
1
Not at all
2
3
4
5
Perfectly
4
Q

Which one of the following best describes the definition of crime?

A) Immoral behaviour

B) Public wrongs

C) Public morals

A

B) Public wrongs

(This is the correct answer. Public wrongs are society’s interpretation of right and wrong behaviour)

How well did you know this?
1
Not at all
2
3
4
5
Perfectly
5
Q

Which one of the following best describes the objectives of criminal law?

A) Resolve disputes between the parties

B) To sue the other party

C) To punish and deter

A

C) To punish and deter

(This is the correct answer. These are two possible objectives of criminal law, others include rehabilitation and protection of the public)

How well did you know this?
1
Not at all
2
3
4
5
Perfectly
6
Q

Which of the following is not a crime?

A) Extra marital sex

B) Inflicting grievous bodily harm

C) Marital rape

A

A) Extra marital sex

(Correct- this is not a crime. We used this example to illustrate the difference between what some people might consider immoral and a crime)

How well did you know this?
1
Not at all
2
3
4
5
Perfectly
7
Q

Which one of the following best describes what criminal liability is concerned with?

A) Sentencing offenders

B) Criminal offences and legal analysis

C) The criminal justice system

D) Allegation, trial and conviction

A

B) Criminal offences and legal analysis

(This is the correct answer. Criminal liability is concerned with identifying criminal offences then applying knowledge of the elements of criminal offences to the specific facts)

How well did you know this?
1
Not at all
2
3
4
5
Perfectly
8
Q

Which of the following best describes the usual burden and standard of proof in criminal law?

A) The burden of proof is on the defence and the standard is beyond reasonable doubt

B) The burden of proof is on the defence and the standard is on the balance of probabilities

C) The burden of proof is on the prosecution and the standard is beyond reasonable doubt

D) The burden of proof is on the prosecution and the standard is on the balance of probabilities

E) The burden of proof is on the prosecution and the standard is beyond doubt

A

C) The burden of proof is on the prosecution and the standard is beyond reasonable doubt

(Correct. On rare occasions (e.g. the defence of diminished responsibility) the burden shifts to the defence and when this happens the standard is on the balance of probabilities)

How well did you know this?
1
Not at all
2
3
4
5
Perfectly
9
Q

Can the same action result in criminal and civil liability?

A) No

B) Yes

A

B) Yes

(Correct- the example we gave was that the crime of battery can also be a tort of trespass to the person)

How well did you know this?
1
Not at all
2
3
4
5
Perfectly
10
Q

Which one of the following elements is not part of the criminal liability equation?

A) Absence of a valid defence

B) Actus reus

C) Motive

D) Mens rea

A

C) Motive

(This is the correct answer. Motive is not required for criminal liability (Chandler v DPP))

How well did you know this?
1
Not at all
2
3
4
5
Perfectly
11
Q

The Theft Act 1968, s 1 provides:

‘A person is guilty of theft if he dishonestly appropriates property belonging to another with the intention of permanently depriving the other of it.’

What is the actus reus of the offence of theft?

A) Property, belonging to another

B) Dishonestly, appropriates, property

C) Appropriates, property, belonging to another

D) Dishonestly, intention of permanently depriving

E) Appropriates, property, intention of permanently depriving

A

C) Appropriates, property, belonging to another

(This is the correct answer. These actus reus elements all have specific meanings defined in statute and case law)

How well did you know this?
1
Not at all
2
3
4
5
Perfectly
12
Q

The Criminal Damage Act 1971, s 1(1) states:

‘A person who without lawful excuse destroys or damages any property belonging to another intending to destroy or damage any such property or being reckless as to whether any such property would be destroyed or damaged shall be guilty of an offence.’

What is the actus reus of the offence of basic criminal damage?

A) Intending or being reckless as to destroying or damaging property belonging to another

B) Destroys, damages, property, without lawful excuse

C) Intending or being reckless

D) Destroys, damages, property, belonging to another, without lawful excuse

E) Destroys, damages, property

A

D) Destroys, damages, property, belonging to another, without lawful excuse

(This is the correct answer. These actus reus elements all have specific meanings defined in statute and case law)

How well did you know this?
1
Not at all
2
3
4
5
Perfectly
13
Q

Which of the following is the definition of a result crime?

A) Require a legal obligation to act which if breached could result in criminal liability

B) Require the need for some particular surrounding circumstance

C) Require certain acts to have been committed

D) Require a guilty action or omission

E) Require that the conduct of the defendant causes a particular consequence

A

E) Require that the conduct of the defendant causes a particular consequence

(This is the correct answer. Examples of result crimes include murder and assault occasioning actual bodily harm)

How well did you know this?
1
Not at all
2
3
4
5
Perfectly
14
Q

Which of the following is the test of factual causation?

A) The defendant’s act must be the ‘substantial’ cause of the prohibited harm

B) The defendant’s act must be a cause which is more than de minimis, more than minimal

C) The consequence must be caused by the defendant’s culpable act

D) The defendant’s act need not be the only cause of the prohibited consequence

E) ‘But for’ the acts or omissions of the defendant, the relevant consequence would not have occurred in the way that it did

A

E) ‘But for’ the acts or omissions of the defendant, the relevant consequence would not have occurred in the way that it did

(This is the correct answer. Sometimes this is referred to as the ‘but for’ test of factual causation)

How well did you know this?
1
Not at all
2
3
4
5
Perfectly
15
Q

Which one of the following is a key authority for the principle of legal causation that the defendant’s act must be the ‘substantial’ cause of the prohibited harm?

A) R v Benge

B) R v Marchant

C) R v Hughes

D) R v Dyson

E) R v White

A

C) R v Hughes

(This is the correct answer. The case clarified that the defendant’s act need not be the only or the principal cause, just more than de minimis)

How well did you know this?
1
Not at all
2
3
4
5
Perfectly
16
Q

Which one of the following is a key authority for the test of factual causation?

A) R v Hughes

B) R v Cato

C) R v Cheshire

D) R v Benge

E) R v White

A

E) R v White

(This is the correct answer. This case is the key authority for the ‘but for’ case)

How well did you know this?
1
Not at all
2
3
4
5
Perfectly
17
Q

Result crimes relate to what aspect of the criminal liability equation?

A) Absolute liability

B) Actus reus

C) Strict liability

D) Mens rea

E) Absence of a valid defence

A

B) Actus reus

(This is the correct answer. Whether the defendant has caused a particular result is a question of actus reus)

How well did you know this?
1
Not at all
2
3
4
5
Perfectly
18
Q

The defendant punches the victim and while most people would have sustained a bruise, the victim dies due to having brittle bones.

Which one of the following describes the legal causation issue?

A) Acts of the victim

B) Acts of a third party

C) Thin skull rule

D) Poor medical treatment

E) Acts of the defendant

A

C) Thin skull rule

(This is the correct answer. The seriousness of an attack escalating due to the victim’s pre-existing medical condition is covered by the thin skull rule (R v Blaue))

How well did you know this?
1
Not at all
2
3
4
5
Perfectly
19
Q

Which of the following correctly describes whether acts of a third party will break the chain of causation?

A) Acts of a third party can break the chain of causation when free

B) Acts of a third party cannot break the chain of causation

C) Acts of a third party can break the chain of causation when free, deliberate and informed

D) Acts of a third party can break the chain of causation when free and deliberate

A

C) Acts of a third party can break the chain of causation when free, deliberate and informed

(This is the correct answer. This reflects the legal position following the case of R v Pagett)

How well did you know this?
1
Not at all
2
3
4
5
Perfectly
20
Q

A woman disagrees with a man. Enraged, the woman grabs a heavy object and uses it to hit the man several times with full force to the head. The man is taken to hospital for treatment. The doctor misreads the man’s notes and administers the wrong medication. The man suffers an allergic reaction and dies from his injuries.

Which of the following statements best explains the woman’s actus reus?

A) The woman may satisfy the actus reus, as the doctor’s misreading of the notes were not so potent in causing the man’s death and so independent of the woman’s actions that it breaks the chain of causation

B) The woman may not satisfy the actus reus, as the doctor’s misreading of the notes was so potent in causing the man’s death and so independent of the woman’s actions that it breaks the chain of causation

C) The woman may not satisfy the actus reus, as he was not aware of the man’s medical condition when he attacked him which breaks the chain of causation

D) The woman may not satisfy the actus reus, as the man’s medical condition means that he died when an ordinary person would have survived which breaks the chain of causation

E) The woman will satisfy the actus reus, as the courts never consider the chain of causation broken due to negligent medical treatment such as the doctor misreading the man’s notes

A

A) The woman may satisfy the actus reus, as the doctor’s misreading of the notes were not so potent in causing the man’s death and so independent of the woman’s actions that it breaks the chain of causation

(This is the correct answer, as the man dies from the injuries the woman inflicted. The doctor giving the man the wrong medication does not render the woman’s actions insignificant, R v Cheshire)

How well did you know this?
1
Not at all
2
3
4
5
Perfectly
21
Q

Which of the following best describes whether you can you be criminally liable for a failure to act?

A) You cannot be criminally liable for a failure to act

B) You cannot generally be criminally liable for a failure to act unless you have a legal duty to act

C) You can be criminally liable for a failure to act

A

B) You cannot generally be criminally liable for a failure to act unless you have a legal duty to act

(Correct. This reflects the general rule in R v Smith and the legal duties to act defined by common law and statute)

How well did you know this?
1
Not at all
2
3
4
5
Perfectly
22
Q

A woman starts to look after her elderly uncle who is bed-bound and unable to feed himself. The woman goes on holiday and forgets to feed her uncle and he dies of starvation.

Which one of the following best describes the legal duty to act the woman may be under?

A) Voluntarily assuming responsibility

B) Creating a dangerous situation

C) Special relationship

D) Public office

E) Contract

A

A) Voluntarily assuming responsibility

(This is the correct answer. The legal duty of voluntarily assuming responsibility may arise in this case, see R v Instan and R v Stone & Dobinson cases)

How well did you know this?
1
Not at all
2
3
4
5
Perfectly
23
Q

A parent fails to feed her child. The child dies of starvation.

Which one of the following best describes the legal duty to act the parent may be under?

A) Creating a dangerous situation

B) Voluntary assumption of responsibility

C) Contract

D) Special relationship

E) Public office

A

D) Special relationship

(Correct. This reflects the position in common law and statute)

How well did you know this?
1
Not at all
2
3
4
5
Perfectly
24
Q

Which of the following describes the concept of mens rea?

A) Mens rea relates to the defendant’s guilty failure to act

B) Mens rea relates to the defendant’s guilty action

C) Mens rea relates to the defendant’s guilty mind

A

C) Mens rea relates to the defendant’s guilty mind

(Correct. The defendant’s guilty mind could be via intention or recklessness for example)

How well did you know this?
1
Not at all
2
3
4
5
Perfectly
25
Q

The Theft Act 1968, s 1 provides:

‘A person is guilty of theft if he dishonestly appropriates property belonging to another with the intention of permanently depriving the other of it.’

What is the mens rea of the offence of theft?

A) Property, belonging to another

B) Dishonestly, intention of permanently depriving

C) Appropriates, property, belonging to another

D) Appropriates, property, intention of permanently depriving

E) Dishonestly, appropriates, property

A

B) Dishonestly, intention of permanently depriving

(This is the correct answer. These mens rea elements all have specific meanings defined in statute and case law)

How well did you know this?
1
Not at all
2
3
4
5
Perfectly
26
Q

The Criminal Damage Act 1971, s 1(1) states:

‘A person who without lawful excuse destroys or damages any property belonging to another intending to destroy or damage any such property or being reckless as to whether any such property would be destroyed or damaged shall be guilty of an offence.’

What is the mens rea of the offence of basic criminal damage?

A) Destroys, damages, property

B) Intending or being reckless as to destroying or damaging property belonging to another

C) Without lawful excuse, destroys, damages, property

D) Intending or being reckless

E) Destroys, damages, property, belonging to another

A

B) Intending or being reckless as to destroying or damaging property belonging to another

(This is the correct answer. The mens rea elements of basic criminal damage are further defined in case law)

How well did you know this?
1
Not at all
2
3
4
5
Perfectly
27
Q

Which one of the following statements is best definition of mens rea?

A) Mens rea is when the defendant acts with motive

B) Mens rea is when the defendant is morally guilty

C) Mens rea is the defendant’s guilty mind

D) Mens rea is the defendant’s guilty act or omission

E) Mens rea is when the defendant desires the consequences of his acts

A

C) Mens rea is the defendant’s guilty mind

(This is the correct answer, it can mean any of the mental elements required for a particular crime)

How well did you know this?
1
Not at all
2
3
4
5
Perfectly
28
Q

Which one of the following statements is the best definition of when a defendant intends a result?

A) A defendant intends a result when he desires it

B) A defendant intends a result when he is malicious

C) A person has direct intent if they have a reason for their actions

D) A defendant intends a result when it is his motive

E) A defendant intends a result when it is the purpose of his act

A

E) A defendant intends a result when it is the purpose of his act

(This is the correct answer according to the case of R v Moloney which suggests that the ordinary meaning of intention should be used)

How well did you know this?
1
Not at all
2
3
4
5
Perfectly
29
Q

What is the correct definition to be given to a jury regarding oblique intention?

A) The jury are not entitled to find oblique intention unless they are sure that death or serious injury was a virtual certainty as a result of the defendant’s action and the defendant appreciated that

B) The jury are not entitled to infer oblique intention unless the event is a natural consequence of the defendant’s voluntary act and the defendant foresees it as such

C) The jury are entitled to find oblique intent when the defendant foresees an event occurring

D) The jury are entitled to find oblique intent when the event is a natural consequence of the defendant’s voluntary act

E) The jury are not entitled to infer oblique intention unless they are sure that death or serious injury was a virtual certainty as a result of the defendant’s action and the defendant appreciated that

A

A) The jury are not entitled to find oblique intention unless they are sure that death or serious injury was a virtual certainty as a result of the defendant’s action and the defendant appreciated that

(This is the correct answer, according to the case of R v Woollin)

How well did you know this?
1
Not at all
2
3
4
5
Perfectly
30
Q

Which part of the criminal liability equation does recklessness relate to?

A) Recklessness is a concept associated with specific intent

B) Recklessness is a concept associated with oblique intent

C) Recklessness is a concept associated with the actus reus

D) Recklessness is a concept associated with the mens rea

A

D) Recklessness is a concept associated with the mens rea

(This is the correct answer, it is one of the ways in which the prosecution can prove the mens rea of a crime of basic intent)

How well did you know this?
1
Not at all
2
3
4
5
Perfectly
31
Q

Which of the following cases contains the current test of recklessness?

A) R v Maloney

B) R v Woollin

C) R v Cunningham

D) R v G

A

D) R v G

How well did you know this?
1
Not at all
2
3
4
5
Perfectly
32
Q

Which of the following best describes the concept of recklessness?

A) A person acts recklessly when he is aware of a risk, goes ahead anyway and it is unreasonable by the standards an ordinary person to take that risk

B) A person acts recklessly when he is told not to do something but decides nevertheless to embark on that course of action

C) A person acts recklessly when he is aware of a risk, goes ahead anyway and in the circumstances known to him, it was an unreasonable risk to take

A

C) A person acts recklessly when he is aware of a risk, goes ahead anyway and in the circumstances known to him, it was an unreasonable risk to take

(Correct. This reflects the test of recklessness in R v G)

How well did you know this?
1
Not at all
2
3
4
5
Perfectly
33
Q

What is the name of the principle that the defendant must have the relevant mens rea for the offence at the precise moment when D commits the actus reus?

A) Continuing act

B) Coincidence

C) One transaction

A

B) Coincidence

(Correct. The courts have interpreted this principle widely using the continuing act theory and the one transaction principle)

How well did you know this?
1
Not at all
2
3
4
5
Perfectly
34
Q

Which of the following statements correctly describes the operation of the doctrine of transferred malice?

A) The doctrine of transferred malice operates to allow the mens rea against X to be transferred and joined with the actus reus that causes the harm to Y

B):The doctrine of transferred malice operates to allow the suspect to be charged with an offence that he did not intend to do

C) The doctrine of transferred malice operates to allow the actus reus against X to be transferred and joined with the mens rea of the offence that relates to harm suffered by Y

A

A) The doctrine of transferred malice operates to allow the mens rea against X to be transferred and joined with the actus reus that causes the harm to Y

(Correct. This reflects the operation of the doctrine as described in key cases such as R v Latimer)

How well did you know this?
1
Not at all
2
3
4
5
Perfectly
35
Q

In which one of these scenarios would the defendant be held criminally liable?

A) A defendant does not know they are breaking the law

B) A defendant mistakenly believes an umbrella is theirs

C) A defendant makes a mistake about civil law negating the mens rea of the offence

A

A) A defendant does not know they are breaking the law

(Correct. Ignorance of the law is no excuse, see the case of Bailey for further details)

How well did you know this?
1
Not at all
2
3
4
5
Perfectly
36
Q

Which of the following is a special and partial defence?

A) Intoxication

B) Consent

C) Loss of control

D) Self-defence

A

C) Loss of control

(Correct. Loss of control is a special defence because it is available for the charge of murder only. Loss of control is a partial defence because if successful, D will be convicted of voluntary manslaughter rather than murder)

How well did you know this?
1
Not at all
2
3
4
5
Perfectly
37
Q

When can the defence of self-defence be used?

A) Protection of yourself

B) Protection of yourself, another and/ or property

C) Protection of yourself and/ or another

A

B) Protection of yourself, another and/ or property

(Correct. If successful the defendant will be acquitted)

How well did you know this?
1
Not at all
2
3
4
5
Perfectly
38
Q

Which question will the court ask to establish whether an involuntarily intoxicated defendant can avoid criminal liability?

A) Would D have foreseen the risk of harm if sober?

B) Did D form the mens rea even though intoxicated?

C) Was D intoxicated when committing the offence?

A

B) Did D form the mens rea even though intoxicated?

(Correct. This is the question the court will ask in situations where D was drugged without consent. See the case of Kingston for example)

How well did you know this?
1
Not at all
2
3
4
5
Perfectly
39
Q

A solider lawfully kills an enemy in battle.

What criminal liability is the solider likely to have, if any?

A) No criminal liability

B) Voluntary manslaughter

C) Murder

A

A) No criminal liability

(Correct. The solider has not fulfilled one of the actus reus elements of murder- to ‘unlawfully’ kill another human being)

How well did you know this?
1
Not at all
2
3
4
5
Perfectly
40
Q

A disgruntled ex-employee feels he is owed money from his former employer and wants to make her pay. The ex-employee goes to his former place of employment and stabs the manager with his knife, killing her.

What criminal liability is the ex-employee likely to have, if any?

A) No criminal liability

B) Murder

C) Voluntary manslaughter

A

B) Murder

(Correct. On the face of it, it looks like the ex-employee has fulfilled the actus reus and mens rea of murder)

How well did you know this?
1
Not at all
2
3
4
5
Perfectly
41
Q

A homeowner who had been previously burgled, suffers from paranoia and post traumatic stress disorder (which will be recognised medical conditions for the purposes of diminished responsibility). The homeowner, who now believes that everyone that comes onto his land is an intruder, shoots a delivery person in the back of the head as they are walking back to their van having dropped off a package. The delivery person dies.

Self-defence is only available if the homeowner uses force that is not grossly disproportionate.

What criminal liability is the homeowner likely to have, if any?

A) Murder

B) Voluntary manslaughter

C) No criminal liability

A

B) Voluntary manslaughter

(Correct. On the face of it, the homeowner has the actus reus and mens rea of murder. As the homeowner suffers from paranoia and post traumatic stress disorder, this suggests he may be able to rely on the partial defence of diminished responsibility. Self-defence is unlikely to be available in this scenario as the homeowner has acted with grossly disproportionate force)

How well did you know this?
1
Not at all
2
3
4
5
Perfectly
42
Q

Which one of the following statements correctly defines malice aforethought?

A) Malice aforethought means an intention to cause grievous bodily harm

B) Malice aforethought means intentionally or recklessly cause the death of another

C) Malice aforethought means an intention to kill

D) Malice aforethought means an intention to kill or an intention to cause grievous bodily harm

A

D) Malice aforethought means an intention to kill or an intention to cause grievous bodily harm

(This is the correct answer. The case of R v Vickers clarified that the mens rea for murder, malice aforethought, can mean express or implied malice)

How well did you know this?
1
Not at all
2
3
4
5
Perfectly
43
Q

Which of the following best describes the definition of murder?

A) Unlawful killing of a human being under the Queen’s Peace with malice aforethought

B) Intentionally killing another person

C) Unlawful killing of a human being under the Queen’s Peace

D) Unlawful killing

A

A) Unlawful killing of a human being under the Queen’s Peace with malice aforethought

(This is the correct answer. This is the definition of murder that Lord Coke provided)

How well did you know this?
1
Not at all
2
3
4
5
Perfectly
44
Q

In which of the following situations is it most likely to be unlawful to kill another human being?

A) Advancement of justice

B) Killing for revenge

C) Killing enemy soldiers in battle

D) Self-defence

A

B) Killing for revenge

(This is the correct answer. This is not one of the situations in which it can be lawful to kill another human being)

How well did you know this?
1
Not at all
2
3
4
5
Perfectly
45
Q

To be criminally liable for murder you must kill a human being.

Which of the following are not considered a human being?

A) A person born alive and capable of independent life

B) A child fully expelled from the mother’s body but still attached by the umbilical cord

C) A corpse

D) A child fully expelled from the mother’s body and born alive

A

C) A corpse

(This is the correct answer. It is not possible to murder a corpse. See also the case of AG-Ref (No 3 of 1994) [1998] AC 245 for another case in which there was held to be no murder)

How well did you know this?
1
Not at all
2
3
4
5
Perfectly
46
Q

If someone is acting under diminished responsibility when they unlawfully kill another human being under the Queen’s peace with intention to kill, what will the defendant be convicted of?

A) Voluntary manslaughter

B) Involuntary manslaughter

C) Diminished responsibility

D) Murder

A

A) Voluntary manslaughter

(Correct, this reflects the position in section 2(3) Homicide Act 1957 (as amended))

How well did you know this?
1
Not at all
2
3
4
5
Perfectly
47
Q

Which of the following best describes the sentencing powers available to a judge in respect of a defendant who is found to have killed under diminished responsibility?

A) Acquittal

B) Sentencing discretion

C) Mandatory life sentence
Correct

A

B) Sentencing discretion

(Correct. The judge will have discretion in sentencing which will mean that mitigating factors can be taken into consideration when sentencing the defendant)

How well did you know this?
1
Not at all
2
3
4
5
Perfectly
48
Q

Medical evidence is helpful to which aspect of the diminished responsibility defence?

A) Substantially impaired D’s ability to do one or more of the following: understand the nature of D’s conduct, form a rational judgment and/ or exercise self-control

B) Abnormality of mental functioning

C) All four aspects

D) Provide an explanation for D’s conduct

E) Recognised medical condition

A

C) All four aspects

(Correct. This reflects the position in R v Brennan that all four of the elements of the defence are concerned with psychiatric matters)

How well did you know this?
1
Not at all
2
3
4
5
Perfectly
49
Q

What type of defence is loss of control?

A) It is a partial defence

B) It is not a defence at all

C) It is more of an exception to murder, rather than a defence

D) It is a full defence

E) It is a general defence

A

A) It is a partial defence

(Correct, this reflects the position in s 54(7) Criminal Justice Act 2009. If the defence of loss of control is successful, the defendant will not be acquitted but convicted of voluntary manslaughter rather than murder)

How well did you know this?
1
Not at all
2
3
4
5
Perfectly
50
Q

Which one of the following is a qualifying trigger for loss of control?

A) The defendant’s loss of control was attributable to a fear of violence

B) The defendant’s loss of control was attributable to a fear of serious violence

C) The defendant’s loss of control was attributable to a fear of assault

D) The defendant’s loss of control was attributable to a fear of actual bodily harm

A

B) The defendant’s loss of control was attributable to a fear of serious violence

(Correct. This reflects the position in s 54 and s 55 Criminal Justice Act 2009)

How well did you know this?
1
Not at all
2
3
4
5
Perfectly
51
Q

For the third element of the loss of control defence, who is the defendant’s conduct compared to?

A) The reaction of the reasonable person in the circumstances of the defendant

B) The reaction of a normal person

C) The reaction of a normal person in the circumstances of the defendant

A

C) The reaction of a normal person in the circumstances of the defendant

(Correct, this broadly reflects the position in s 54(1)(c) Criminal Justice Act 2009)

How well did you know this?
1
Not at all
2
3
4
5
Perfectly
52
Q

Which of the following is correct?

A) D can rely on loss of control if charged with attempted murder

B) D can rely on loss of control if charged with any offence

C) D can rely on loss of control if charged with murder

A

C) D can rely on loss of control if charged with murder

(Correct. D cannot rely on loss of control if D is charged with attempted murder or any other offence)

How well did you know this?
1
Not at all
2
3
4
5
Perfectly
53
Q

Which of the following is a limitation of the loss of control defence?

A) The defence cannot be used if it stems from a desire for revenge

B) The defence cannot be used if D uses violence first

C) The defence cannot be used if it stems from a considered desire for revenge

A

C) The defence cannot be used if it stems from a considered desire for revenge

(Correct. This reflects the position in section 54(4) Coroners and Justice Act 2009)

How well did you know this?
1
Not at all
2
3
4
5
Perfectly
54
Q

When will sexual infidelity prevent a defendant from being able to rely on the anger trigger?

A) When there are other factors that caused D to lose self-control for the purposes of a qualifying trigger along with sexual infidelity

B) When the thing said/ done constituted sexual infidelity

C) When sexual infidelity is in the background

A

B) When the thing said/ done constituted sexual infidelity

(Correct. This reflects the position in section 55(6)(c) CJA 2009 and Clinton)

How well did you know this?
1
Not at all
2
3
4
5
Perfectly
55
Q

A defendant is charged with murder but is intoxicated at the time. When considering the defendant’s criminal liability at what point in your analysis will you take into account the intoxication?

A) Mens rea

B) Unlawful

C) Causation

A

A) Mens rea

(Correct. The question you will ask is even though intoxicated, did D form the mens rea of intention to kill or intention to cause grievous bodily harm?)

How well did you know this?
1
Not at all
2
3
4
5
Perfectly
56
Q

A defendant is a drug addict who kills under a loss of control having been taunted about his addiction by the victim. The defendant was intoxicated at the time. When considering the defendant’s criminal liability at what point in your loss of control analysis will you take into account the addiction?

A) Loss of self-control

B) The normal person test

C) The anger trigger and the normal person test

D) The anger trigger

A

C) The anger trigger and the normal person test

(Correct. D’s drug or alcohol addiction can be taken into account in assessing the magnitude of the qualifying anger trigger if D was taunted about the addiction. If D is addicted to drugs or alcohol this will be a characteristic given to the normal person but the normal person will still have normal levels of tolerance and self-restraint and be sober, Asmelash)

How well did you know this?
1
Not at all
2
3
4
5
Perfectly
57
Q

In which of the following situations will D not be able to rely on diminished responsibility?

A) When D has an abnormality of mental functioning and is voluntarily intoxicated

B) When D’s abnormality of mental functioning arises from alcohol dependency syndrome

C) When D is voluntarily intoxicated and has no abnormality of mental functioning

A

C) When D is voluntarily intoxicated and has no abnormality of mental functioning

(Correct. This represents the position in Dowds)

How well did you know this?
1
Not at all
2
3
4
5
Perfectly
58
Q

Which of the following elements distinguishes murder from involuntary manslaughter?

A) Killing

B) Malice aforethought

C) Human being

D) Unlawful

A

B) Malice aforethought

(Correct. With murder and involuntary manslaughter the defendant unlawfully kills another human being under the Queen’s Peace. However, with murder D unlawfully kills another human being with intention to kill or intention to cause GBH. Malice aforethought is absent from involuntary manslaughter)

How well did you know this?
1
Not at all
2
3
4
5
Perfectly
59
Q

Which of the following will amount to an unlawful act for the purposes of unlawful act manslaughter?

A) A civil act

B) A criminal act

C) A lawful act which becomes unlawful due to negligence or recklessness

D) An omission

A

B) A criminal act

(Correct. The unlawful act cannot be a civil act, R v Franklin. The unlawful act must not be an omission (such would be charged as gross negligence manslaughter), R v Lowe. The unlawful act must be intrinsically unlawful, Andrews v DPP)

How well did you know this?
1
Not at all
2
3
4
5
Perfectly
60
Q

What is the test for whether the unlawful act was dangerous for the purposes of unlawful act manslaughter?

A) Sober and reasonable people would inevitably recognise must subject the other person to, at least, the risk of some harm, albeit not serious harm

B) Sober and reasonable people would inevitably recognise must subject the other person to serious harm

C) Reasonable people would inevitably recognise must subject the other person to, at least, the risk of some harm

D) Reasonable people would inevitably recognise must subject the other person to serious harm

A

A) Sober and reasonable people would inevitably recognise must subject the other person to, at least, the risk of some harm, albeit not serious harm

(Correct. This reflects the test given in R v Church, the others were incorrect)

How well did you know this?
1
Not at all
2
3
4
5
Perfectly
61
Q

Which one of the following best describes the type(s) of involuntary manslaughter that can be committed by a positive act?

A) Unlawful act manslaughter

B) Gross negligence manslaughter

C) Unlawful act manslaughter and gross negligence manslaughter

A

C) Unlawful act manslaughter and gross negligence manslaughter

(Correct. Unlawful act manslaughter cannot be committed by an omission, R v Lowe. However, gross negligence manslaughter can be committed by a positive act or an omission)

How well did you know this?
1
Not at all
2
3
4
5
Perfectly
62
Q

What is the test for whether there is a risk of death for the purposes of gross negligence manslaughter?

A) Was there a serious risk of death which was obvious either presently or would have become apparent on further investigation?

B) Was there an obvious and serious risk not merely of injury or even serious injury, but of death?

C) Was there an obvious and serious risk of either injury, serious injury or death?

A

B) Was there an obvious and serious risk not merely of injury or even serious injury, but of death?

(Correct. This reflects the wording given in Singh and approved in Misra and **Srivastava. **In Rose the court stated that an obvious risk was a present risk which was clear and unambiguous not one which might become apparent on further investigation)

How well did you know this?
1
Not at all
2
3
4
5
Perfectly
63
Q

Which of the following is correct regarding whether the breach was sufficiently serious to constitute gross negligence for the purposes of gross negligence manslaughter?

A) A single devastating act can be grossly negligent

B) The defendant must have a particular state of mind- D must either have seen a clear risk of death or not cared about the risk of death

C) There cannot be gross negligence by the defendant if others are also responsible for the circumstances leading up to death

A

A) A single devastating act can be grossly negligent

(Correct. This is reflected in the case of Adomako.
Although the defendant’s state of mind is not irrelevant (Litchfield), there is no requirement for any particular mental state, **Adomako. **There can be gross negligence by the defendant if others are also responsible for the circumstances leading up to death, Prentice and Sullman)

How well did you know this?
1
Not at all
2
3
4
5
Perfectly
64
Q

A man believes the waiter is looking at his girlfriend. The man confronts the waiter saying, ‘when I finish with you, you will not be able to look at another girl again’. The man punched the waiter hard in the face. The waiter slipped and hit his head on a nearby table. The waiter died from his injuries.

Which of the following offences is the man most likely to be criminally liable for?

A) Unlawful act manslaughter

B) Gross negligence manslaughter

C) Voluntary manslaughter by diminished responsibility

D) Murder

E) Voluntary manslaughter by loss of control

A

D) Murder

(This is the correct answer. The man causes the waiter’s death by the punch. It is the man’s aim to cause grievous bodily harm by the hard punch to the face)

How well did you know this?
1
Not at all
2
3
4
5
Perfectly
65
Q

A man was queuing to buy the latest smartphone. As he neared the front of the queue, a woman walked past him. As he thought the woman was going to jump the queue, he pushed her and shouted, ‘Hey we have a queue here!’ The woman fell off the kerb and she bumped her head on the road. She subsequently died from her injuries.

Which of the following offences is the man most likely to be criminally liable for?

A) Voluntary manslaughter by diminished responsibility

B) Voluntary manslaughter by loss of control

C) Murder

D) Gross negligence manslaughter

E) Unlawful act manslaughter

A

E) Unlawful act manslaughter

(This is the correct answer. The man did not aim to kill her or cause her serious harm. The push is a battery which is the unlawful act)

How well did you know this?
1
Not at all
2
3
4
5
Perfectly
66
Q

A zookeeper is showing some poisonous snakes to a group of school children. During the presentation the zookeeper’s phone rings. Distracted, the zookeeper fails to close the lid to the tank properly. The zookeeper notices that a dangerous snake is no longer in the tank but not wishing to alarm the children he says nothing. A girl starts screaming. The zookeeper sees that the missing snake has crawled onto the girl’s lap and bitten her on the arm, drawing blood. The girl dies later from her wound.

Which of the following offences is the zookeeper most likely to be criminally liable for?

A) Voluntary manslaughter by loss of control

B) Gross negligence manslaughter

C) Unlawful act manslaughter

D) Voluntary manslaughter by diminished responsibility

E) Murder

A

B) Gross negligence manslaughter

(This is the correct answer. The zookeeper caused the girl’s death by an omission when under a legal duty to act under contract and following the creation of a dangerous situation)

How well did you know this?
1
Not at all
2
3
4
5
Perfectly
67
Q

Which of the following statements about the mens rea of battery are correct?

A) The offence of battery is a specific intent offence

B) The offence of battery can only be committed recklessly

C) The offence of battery requires that the defendant apply unlawful force to another intentionally or recklessly

A

C) The offence of battery requires that the defendant apply unlawful force to another intentionally or recklessly

(Correct. This is the mens rea of battery, R v Venna. As the mens rea of battery states it can be committed recklessly, battery is a basic intent offence)

How well did you know this?
1
Not at all
2
3
4
5
Perfectly
68
Q

Where is assault defined?

A) Assault is a common law offence

B) Section 39 Criminal Justice Act 1988 codifies the common law definition of assault

C) Assault is a statutory offence, defined in section 39 Criminal Justice Act 1988

A

A) Assault is a common law offence

(Correct. It is defined in the Fagan v MPC case. The penalties and procedure are set out in section 39 Criminal Justice Act 1988)

How well did you know this?
1
Not at all
2
3
4
5
Perfectly
69
Q

For the purpose of an assault, what is meant by personal violence?

A) The victim must apprehend psychological harm

B) The victim must apprehend physical violence

C) The victim must apprehend physical and/ or psychological harm

D) The defendant must have made physical contact with the victim

A

B) The victim must apprehend physical violence

(Correct. For an assault, the victim must apprehend physical violence. The view that personal violence could include psychological harm was expressly rejected by Lord Hope in the House of Lords in the Ireland case. If the defendant has made contact with the victim, battery is the more appropriate offence)

How well did you know this?
1
Not at all
2
3
4
5
Perfectly
70
Q

What is the mens rea of section 20 Offences Against the Person Act 1861, malicious wounding or inflicting grievous bodily harm?

A) Intention or recklessness as to wounding or inflicting grievous bodily harm

B) Intention or recklessness as to causing some harm

C) Intention to cause serious harm

D) Intention or recklessness as to causing harm

A

B) Intention or recklessness as to causing some harm

(Correct. This was confirmed in R v Savage, Parmenter)

How well did you know this?
1
Not at all
2
3
4
5
Perfectly
71
Q

What is the mens rea of assault occasioning actual bodily harm?

A) The mens rea for the assault or battery and intention or recklessness as to causing actual bodily harm

B) The mens rea for the assault or battery

C) Intention or recklessness as to causing actual bodily harm

A

B) The mens rea for the assault or battery

(Correct. No mens rea is required for the actual bodily harm, R v Savage, R v Parmenter)

How well did you know this?
1
Not at all
2
3
4
5
Perfectly
72
Q

What is the mens rea of section 20 Offences Against the Person Act 1861, malicious wounding or inflicting grievous bodily harm?

A) Intention or recklessness as to causing harm

B) Intention or recklessness as to causing some harm

C) Intention or recklessness as to wounding or inflicting grievous bodily harm

D) Intention to cause serious harm

A

B) Intention or recklessness as to causing some harm

(Correct. This was confirmed in R v Savage, Parmenter)

How well did you know this?
1
Not at all
2
3
4
5
Perfectly
73
Q

What is the mens rea of section 18 Offences Against the Person Act 1861, malicious wounding or causing grievous bodily harm with intent?

A) Intention to wound or inflict grievous bodily harm

B) Intention to cause grievous bodily harm

C) Intention to cause some harm

D) Intention or recklessness as to causing serious harm

A

B) Intention to cause grievous bodily harm

(Correct. Grievous bodily harm means serious harm, Saunders)

How well did you know this?
1
Not at all
2
3
4
5
Perfectly
74
Q

A girl picks up a gun and points it at a boy’s head. The girl and the boy know that the gun is not loaded.

Which one of the following statements best describes the legal position?

A) The girl has no criminal liability as has not committed either an assault or a battery

B) The girl is potentially criminally liable for an assault

C) The girl is potentially criminally liable for a battery

A

A) The girl has no criminal liability as has not committed either an assault or a battery

(Correct. The girl has not threatened unlawful violence on the boy nor touched him without consent, so does not fulfil the definitions of assault or battery)

How well did you know this?
1
Not at all
2
3
4
5
Perfectly
75
Q

Which one of the following best describes the mens rea of s 47 Offences Against the Person Act 1861?

A) An intention or recklessness as to applying unlawful force to another

B) There is no mens rea requirement

C) An intention or recklessness as to applying unlawful force to another or an intention or recklessness as to causing the victim to apprehend immediate and unlawful personal violence

D) An intention or recklessness as to causing the victim to apprehend immediate and unlawful personal violence

A

C) An intention or recklessness as to applying unlawful force to another or an intention or recklessness as to causing the victim to apprehend immediate and unlawful personal violence

(Correct. No mens rea is required for the actual bodily harm element)

How well did you know this?
1
Not at all
2
3
4
5
Perfectly
76
Q

Which of the following correctly describes the mens rea of s 20 Offences Against the Person Act 1861?

A) Intention to cause serious harm

B) Intention or recklessness as to causing serious harm

C) Intention to cause some harm

D) Intention or recklessness as to causing some harm

A

D) Intention or recklessness as to causing some harm

(Correct. Intention to cause serious harm is the mens rea of s 18 OAPA 1861)

How well did you know this?
1
Not at all
2
3
4
5
Perfectly
77
Q

If a person commits an offence of basic arson, how will the charge read?

A) Section 1(1) Criminal Damage Act 1971

B) Section 1(3) Criminal Damage Act 1971

C) Section 1(1) and s 1(3) Criminal Damage Act 1971

A

C) Section 1(1) and s 1(3) Criminal Damage Act 1971

(Correct. Basic criminal damage is charged under s 1(1) and any criminal damage by fire is charged as arson under s 1(3))

How well did you know this?
1
Not at all
2
3
4
5
Perfectly
78
Q

Which of the following is property for the purposes of the Criminal Damage Act 1971?

A) Mushrooms growing wild

B) Tamed animals

C) Information

A

B) Tamed animals

(Correct. This reflects the wording of s 10(1)(a) which states that property includes wild creatures that have been tamed. Mushrooms growing wild are excluded by s 10(1)(b) and information was held not to be property for the purposes of the CDA 1971 in the case of R v Whitely)

How well did you know this?
1
Not at all
2
3
4
5
Perfectly
79
Q

Which of the following is not defined in the Criminal Damage Act 1971?

A) Damage

B) Belonging to another

C) Property

A

A) Damage

(Correct. Damage is defined in case law but property and belonging to another are defined in sections 10(1) and 10(2) respectively)

How well did you know this?
1
Not at all
2
3
4
5
Perfectly
80
Q

Mike is very proud of his car. He believes that its paintwork is being damaged by pollutants caused by a nearby factory. Mike feels that the only way to stop any further damage to his car he must get rid of the factory, so he burns it down.

Which defence, if any, could you argue on behalf of Mike?

A) Section 5(2)(a)- Mike believes that the owner would have consented

B) None of the lawful excuse defences are arguable

C) Section 5(2)(b)- Mike believes that he must act to protect property

A

C) Section 5(2)(b)- Mike believes that he must act to protect property

(Correct. Arguably, Mike honestly believed that his car was in immediate need of protection from the factory and that the means of protection adopted were reasonable to ensure no further damage. Burning down the factory was objectively capable of protecting the property)

How well did you know this?
1
Not at all
2
3
4
5
Perfectly
81
Q

Which of the following requirements of the section 5(2)(b) lawful excuse defence is objective?

A) D must believe that the means of protection adopted are reasonable

B) D must believe that the property was in immediate need of protection

C) The damage caused by D must be capable of protecting the property

D) D must act to protect property

A

C) The damage caused by D must be capable of protecting the property

(Correct. This was outlined in the case of R v Hunt and confirmed in later cases)

How well did you know this?
1
Not at all
2
3
4
5
Perfectly
82
Q

Mike is very proud of his car. He believes that its paintwork is being damaged by pollutants caused by a nearby factory. Mike feels that the only way to stop any further damage to his car he must get rid of the factory, so he burns it down.

Which defence, if any, could you argue on behalf of Mike?

A) None of the lawful excuse defences are arguable

B) Section 5(2)(a)- Mike believes that the owner would have consented

C) Section 5(2)(b)- Mike believes that he must act to protect property

A

C) Section 5(2)(b)- Mike believes that he must act to protect property

(Correct. Arguably, Mike honestly believed that his car was in immediate need of protection from the factory and that the means of protection adopted were reasonable to ensure no further damage. Burning down the factory was objectively capable of protecting the property)

How well did you know this?
1
Not at all
2
3
4
5
Perfectly
83
Q

To which offences do the lawful excuse defences within the Criminal Damage Act 1971 section 5(2) potentially apply?

A) Basic criminal damage and basic arson

B) All criminal damage offences

C) All criminal offences

D) Aggravated criminal damage and aggravated arson

A

A) Basic criminal damage and basic arson

(Correct. This is set out in section 5(1) Criminal Damage Act 1971)

How well did you know this?
1
Not at all
2
3
4
5
Perfectly
84
Q

Which of the following are not required for aggravated criminal damage?

A) Property

B) Destroy/ damage

C) Belonging to another

A

C) Belonging to another

(Correct. Property can belong to another or the defendant for the purposes of aggravated criminal damage or aggravated arson)

85
Q

Which of the following is part of the mens rea of aggravated criminal damage?

A) Life must actually be endangered

B) Danger to life must arise from the damaged property

C) Danger to life must arise from the means of damaging the property

A

B) Danger to life must arise from the damaged property

(Correct. This is the principle illustrated in R v Steer. In R v Steer the danger to life was three bullets fired (the means of damaging the property) but this did not constitute aggravated criminal damage)

86
Q

Do the lawful excuse defences in section 5(2) apply to aggravated criminal damage or aggravated arson?

A) It depends on the circumstances of the case

B) Yes

C) No

A

C) No

(Correct. The lawful excuse defences do not apply to aggravated criminal damage or aggravated arson, s 5(1) states this)

87
Q

How many rights of the owner must be assumed to appropriate property?

A) One

B) Two or more

C) None

A

A) One

(Correct. Any assumption by a person of the rights of an owner amounts to an appropriation (s 3(1) Theft Act 1968) and it need be just one right (R v Morris))

88
Q

Can you appropriate a valid gift?

A) No

B) Yes

A

B) Yes

(Correct- this reflects the position in R v Hinks)

89
Q

Can you appropriate property with the owners consent?

A) Yes

B) No

A

A) Yes

(Correct- this reflects the position in R v Gomez)

90
Q

Karen picks blackberries from the side of the road, makes them into jam and sells the jam in aid of charity. The blackberries capable of being stolen?

A) False

B) True

A

B) True

(Correct. The blackberries are growing wild, but she has picked them for sale, so they are capable of being stolen, see s 4(3) Theft Act 1968. They would belong to the owner of the land, possibly the highways authority - all land is owned by somebody. She may not be liable for theft however as she may not be dishonest)

91
Q

Gloria picks wild mushrooms on her walks through the woods and uses them to cook with in her restaurant. Gloria has stolen the mushrooms.

A) False

B) True

A

B) True

(Correct. Yes (assuming Gloria does not own the woods and is dishonest). The mushrooms are growing wild and she picks them for a commercial purpose, s 4(3) Theft Act 1968)

92
Q

Property can belong to more than one person for the purposes of the Theft Act 1968.

A) False

B) True

A

B) True

(For the purposes of the Theft Act property can belong to the owner, while at the same time belonging to someone having possession or control as per s 5(1). It can belong to the owner, while at the same time belonging to another by virtue of s 5(3) or s 5(4))

93
Q

Helen shares a taxi with her friend Sally, who gets out first. Sally gives Helen money to pay for the taxi and extra money to tip the driver. Helen keeps the money she was given for the tip.

What section of the Theft Act 1968 is required for the property to belong to another?

A) Section 5(1)

B) Section 5(4)

C) Section 5(3)

A

C) Section 5(3)

(Correct. Here the extra money is given by Sally to Helen for a particular purpose, to tip the driver)

94
Q

Dan has left his shoes to be re-heeled at a shop. He takes them from the shop while the assistants back is turned without paying for them.

What section of the Theft Act 1968 is required for the property to belong to another?

A) Section 5(3)

B) Section 5(4)

C) Section 5(1)

A

C) Section 5(1)

(Correct. The shop is in possession or control of the shoes by virtue of section 5(1). This scenario is similar to the Turner (No 2) case where D owed money to a garage for car repairs)

95
Q

S5(4) creates a legal obligation to make restoration of the property got by another’s mistake.

A) True

B) False

A

B) False

(This statement is false. Section 5(4) only applies when a legal obligation to make restoration exists. It then operates to make the property belong to another for the purposes of the Theft Act 1968)

96
Q

Dishonesty is defined in the Theft Act 1968

A) True

B) False

A

B) False

(The Theft Act 1968 does not define dishonesty. However, section 2(1) Theft Act 1968 provides three circumstances of when a defendant will not be dishonest)

97
Q

Which one of the following statements is the most accurate on the test for dishonesty?

A) The only question to ask is- Does the judge consider the defendant’s actions to be dishonest?

B) Two questions need to be asked- (i) What was the defendant’s knowledge and belief as to the facts? (ii) Given that knowledge and those beliefs, was the defendant dishonest by the standards of ordinary decent people?

C) The only question to ask is- Does the defendant consider their actions to be dishonest?

D) The only question to ask is- Would ordinary reasonable people consider the defendant’s actions to be dishonest?

E) Two questions need to be asked- (i) Would ordinary reasonable people consider the actions of the defendant to be dishonest? (ii) Does the defendant realise that ordinary decent people would consider their actions to be dishonest?

A

B) Two questions need to be asked- (i) What was the defendant’s knowledge and belief as to the facts? (ii) Given that knowledge and those beliefs, was the defendant dishonest by the standards of ordinary decent people?

(Correct. This is the test as set out in Ivey v Genting Casinos)

98
Q

The Theft Act 1968 section 2 provides three situations in which an appropriation of property is not to be regarded as dishonest: belief in a right in law, another’s consent and that the owner cannot be discovered by taking reasonable steps.

What type of belief is required by the Theft Act 1968 for a defendant not to be regarded as dishonest?

A) Subjective- the defendant’s belief must be genuinely held

B) Objective- the reasonable person must be capable of believing the defendant

C) Subjective- the defendant’s belief must be reasonably held

A

A) Subjective- the defendant’s belief must be genuinely held

(Correct. This was confirmed by the case of R v Robinson)

99
Q

Kitty takes a carton of milk belonging to Fiona. She intends to replace it later that day. Which of the following best describes her intention to permanently deprive?

A) Kitty takes Fiona’s carton of milk and treats it in a manner which she knows risks its loss

B) Kitty’s intention to return Fiona’s carton of milk with another is still intention to permanently deprive

C) Kitty’ has taken the carton of milk but may not be able to fulfil the condition of its return by replacing it

A

B) Kitty’s intention to return Fiona’s carton of milk with another is still intention to permanently deprive

(Correct. This comes from the case of R v Velumyl on interchangeable property still amounting to an intention to permanently deprive. Kitty does not intend to return the same carton of milk, so she has an intention to permanently deprive under s 1 Theft Act 1968)

100
Q

In which one of the following situations does section 1(1) Theft Act 1968 rather than section 6(1) need to be referred to in order to establish intention to permanently deprive?

A) D takes V’s property with an intention to keep it

B) D takes V’s property and attempts to sell it back to V

C) D takes V’s property and treats it in a manner that risks its loss

D) D borrows V’s property

A

A) D takes V’s property with an intention to keep it

(Correct. This is a simple case where the ordinary meaning of intention to permanently deprive from section 1(1) Theft Act 1968 is appropriate as it is clear on the facts)

101
Q

Which is the wider definition of the term ‘to treat the thing as his own to dispose of regardless of the other’s rights’?

A) Intending to treat it in a manner which risks its loss

B) The dictionary definition of ‘to dispose of’- to deal with definitely: to get rid of; to get done with, finish. To make over by way of sale or bargain, sell’

A

A) Intending to treat it in a manner which risks its loss

(Correct. This definition was provided in the case of R v Fernandes. Auld LJ says intention to permanently deprive is not limited the definition from Potts J in R v Cahill. Potts J defines this phrase as ‘to get rid of; to get done with, finish. To make over by way of sale or bargain, sell.’ Auld L.J. says the definition is wider than this and includes an intention to risk its loss)

102
Q

Can you have a robbery without a theft?

A) No

B) Depends on the circumstances

C) Yes

A

A) No

(Correct. The wording of s 8(1) Theft Act 1968 and the case of R v Robinson makes that clear)

103
Q

How do we decide whether there has been enough force for a robbery?

A) The statute provides a definition of ‘force’ as more than a mere touch

B) The statute provides a negative definition of ‘force’ as not requiring violence

C) The judge decides on the meaning of the word ‘force’ and directs the jury

D) The jury decide by using the ordinary meaning of the word ‘force’

A

D) The jury decide by using the ordinary meaning of the word ‘force’

(Correct. Force is not defined in the Theft Act 1968 and it is a matter of fact for the jury whether the defendant’s actions amount to force, R v Dawson and James)

104
Q

Can force be applied through property?

A) No, as force to detach property cannot count as force on the person

B) Yes, as long as the force against the property caused force against the person

C) Yes, as the acts of a pickpocket are enough force for a robbery where there is very minimal indirect contact and no resistance from the victim

D) No, as the wording of the statute refers to the use of force being on any person

A

B) Yes, as long as the force against the property caused force against the person

(Correct. While force is not defined by the Theft Act 1968, this statement accurately reflects the legal principle from R v Clouden)

105
Q

Abi comes across Dan who is walking his dog, Poppy in the park. Abi scoops up Poppy and holds a knife to Poppy’s throat, saying menacingly ‘Give me your wallet or the dog cops it.’ Dan gives Abi his wallet.

Assume a theft has been committed. Is Abi criminally liable for robbery?

A) No- the timing element is missing

B) Yes- Dan fears a dog will be subjected to force

C) No- the force element is missing

A

C) No- the force element is missing

(Correct. This is not robbery. Although she has stolen Dan’s wallet and threatened force immediately before stealing and in order to steal she hasn’t used force, or put any person in fear of force nor has she sought to put any person in fear of being subjected to force. Making a person (Dan) fear a dog, or indeed another person would be subjected to force isn’t included in the definition)

106
Q

Ben insults Matt so Matt punches Ben in the face. As a result Ben’s wallet falls out of his pocket. Matt picks up the wallet and runs off with it.

Assume a theft has been committed. Is Matt criminally liable for robbery?

A) Yes

B) No- he has not used force in order to steal

C) No- the timing element is missing

A

B) No- he has not used force in order to steal

(Correct. Matt has stolen. He has used force on Ben and has done so immediately before stealing. However, this is not robbery as he has not done so in order to steal)

107
Q

Abdul goes into the local shop and tells Ravi, who is behind the counter, that he knows where Ravi’s mother lives and unless Ravi gives him five packets of cigarettes he will break Ravi’s mother’s leg. Ravi gives Abdul cigarettes.

Assume a theft has been committed. Is Abdul criminally liable for robbery?

A) No- the force element is missing

B) No- the timing element is missing

C) Yes

A

A) No- the force element is missing

(Correct. Abdul has stolen and immediately before and in order to do so threatened force, but there is nobody who is frightened that they will be subjected to force and nobody whom Abdul is seeking to put in fear of being subjected to force)

108
Q

Which of the following offences can make up a s 9(1)(a) burglary but not a s 9(1)(b) burglary?

A) Criminal damage

B) Theft

C) Grievous bodily harm

A

A) Criminal damage

(Correct. This reflects the wording of s 9(1)(b) and s 9(2) Theft Act 1968)

109
Q

At what point in time does a s 9(1)(a) burglary take place?

A) Before entry

B) Upon entry with intention to commit the ulterior offence

C) Once D is inside the building or part of the building

D) Once D is inside and has committed or attempted either GBH or theft

A

B) Upon entry with intention to commit the ulterior offence

(Correct. There is no need for the defendant actually to commit the ulterior offence)

110
Q

When establishing a s 9(1)(b) burglary, which of the following elements requires no mens rea?

A) Knowing or being reckless as to entry as a trespasser

B) Grievous bodily harm

C) Theft

A

B) Grievous bodily harm

(Correct. This is the current law according to R v Jenkins. The inflicting of grievous bodily harm is all that is required, the defendant does not need to intend to inflict grievous bodily harm)

111
Q

When must D have the article with them for the purposes of a section 9(1)(b) aggravated burglary?

A) Any time before leaving the building or part of a building

B) On commission or attempted commission of theft or grievous bodily harm

C) At the point of entry
Correct

A

B) On commission or attempted commission of theft or grievous bodily harm

(Correct. This was confirmed in the case of R v O’Leary for example)

112
Q

What is the maximum sentence for aggravated burglary?

A) Life imprisonment

B) 14 years

C) 10 years

A

A) Life imprisonment

(Correct. The reason given by the Criminal Law Revision Committee at the time for this maximum sentence is that aggravated burglary could be very frightening to anyone in the building and could potentially lead to fatal consequences)

113
Q

A man breaks into a house with the aim of stealing valuables and takes a laptop. He has pepper spray in his pocket. Which of the following offences is the man most likely to be criminally liable for?

A) Theft

B) Burglary

C) No criminal law offence

D) Robbery

E) Aggravated burglary

A

E) Aggravated burglary

(This is the correct answer. The man commits a burglary and at the time he has with him a weapon of offence. Pepper spray is a weapon of offence as it is made for incapacitating a person. See s 10 Theft Act 1968)

114
Q

Felix commits burglary.

He has in the bottom of his rucksack an imitation handgun which he is looking after for a friend, but does not intend to use. The handgun is not capable of being fired.

Is Felix guilty of aggravated burglary and if so how?

A) Felix will be guilty of aggravated burglary as long as he has the rucksack with him during the burglary

B) Felix will not be guilty of aggravated burglary as it is incapable of being fired

C) Felix will not be guilty of aggravated burglary as he does not intend to use it

A

A) Felix will be guilty of aggravated burglary as long as he has the rucksack with him during the burglary

(Correct. A handgun is a firearm, so an imitation handgun is an imitation firearm by virtue of s 10(1)(a) as it will have the appearance of being a firearm. This subsection states that it doesn’t matter that the imitation gun is incapable of being discharged. There is nothing to say that he has to intend to use the imitation firearm)

115
Q

Felix commits burglary.

He has a firework in his pocket.

Is Felix guilty of aggravated burglary and if so how?

A) Felix will not be guilty of aggravated burglary as the firework is not an explosive

B) Felix will not be guilty of aggravated burglary as he does not intend to do anything with the firework during the burglary

C) Felix is guilty of aggravated burglary as the firework is an explosive

A

C) Felix is guilty of aggravated burglary as the firework is an explosive

(Correct. A firework is an explosive by virtue of s 10(1)(c). It is an article manufactured for the purpose of producing a practical effect by explosion. There is no need for him to intend to do anything with the firework during the burglary)

116
Q

Felix commits burglary.

In the kitchen he picks up a knife which he intends to use on anyone who disturbs him.

Is Felix guilty of aggravated burglary and if so how?

A) Felix will only be guilty of aggravated burglary if someone disturbs him and he uses the knife

B) Felix is guilty of aggravated burglary

C) Felix is not guilty of aggravated burglary

A

B) Felix is guilty of aggravated burglary

(Correct. The kitchen knife is a weapon of offence by virtue of s 10(1)(b). It is not made to cause injury or to incapacitate a person, nor is it adapted for use for doing this, but it is intended by the person having it with him for such use. It is intended to be used to injure a person. Intention includes conditional intention i.e. to use the knife in the event that someone disturbs him)

117
Q

Which of the following is the actus reus of fraud by false representation?

A) Dishonesty

B) False representation

C) Intention to make a gain or cause a loss

A

B) False representation

(Correct- this representation can be express or implied, a representation as to fact, law or state of mind and the representation must be untrue or misleading)

118
Q

Overcharging can be a form of fraud by false representation. What other offence could overcharging be if there is a relationship of mutual trust?

A) Fraud by abuse of position

B) Fraud by failure to disclose

C) Deception

A

A) Fraud by abuse of position

(Correct. Although it is acknowledged that fraud by false representation is a more straightforward route to establish liability in this case)

119
Q

How is dishonesty defined for the purposes of the Fraud Act 2006?

A) Section 2(1) Theft Act 1968 negative definitions of dishonesty

B) Ivey v Genting Casinos

C) Section 2(1) Theft Act 1968 negative definitions of dishonesty then Ivey v Genting Casinos

A

B) Ivey v Genting Casinos

(Correct. The negative definitions of dishonesty contained in the Theft Act 1968, s 2(1) apply only to the offence of theft and therefore do not apply to offences under the Fraud Act 2006)

120
Q

What type of legal duty to disclose is a contract of insurance?

A) A duty rising from a fiduciary relationship

B) A duty arising from statute

C) A duty within a transaction of the utmost good faith

A

C) A duty within a transaction of the utmost good faith

(Correct. This was one of the examples provided by the Law Commission in its report that led to the Fraud Act 2006)

121
Q

How many offences of fraud are there?

A) Three

B) Four

C) One

A

C) One

(Correct. There is one offence of fraud in section 1 Fraud Act 2006 but three ways of committing it)

122
Q

Can you be criminally liable for fraud by failure to disclose if there was no gain or loss?

A) No

B) Yes

A

B) Yes

(Correct- the mens rea does not require an actual gain or loss just an intention to make a gain, cause a loss or expose someone to a risk of loss)

123
Q

What section of the Fraud Act 2006 is fraud by abuse of position contained in?

A) Section 4

B) Section 1

C) Section 3

D) Section 2

A

A) Section 4

(Correct- however, the offence of fraud is contained within the Fraud Act 2006, section 1)

124
Q

Who decides whether the defendant occupies a position for the purposes of fraud by abuse of position?

A) The jury alone

B) The judge and the jury have a role in this

C) The judge alone

A

B) The judge and the jury have a role in this

(Correct. The jury will decide on the facts whether the necessary relationship exists for D to have occupied a position in which they are expected to safeguard or not act against the financial interests of another person. However, before the jury decide this, the judge will consider whether the particular facts are capable of giving rise to D occupying a position and may give specific directions to the jury on this point)

125
Q

A man starts visiting his elderly neighbour, helping with her shopping and collecting her pension from the Post Office. How is the man occupying a position for the purposes of fraud by abuse of position?

A) Family relationship

B) Voluntary work

C) Professional relationship

D) Fiduciary relationship

A

B) Voluntary work

(Correct. This relationship would fit into the example given by the Law Commission of ‘voluntary work’)

126
Q

Which of the following statements is the most accurate in relation to the offence of fraud by false representation?

A) The defendant must intend to make a gain or cause loss in property or money terms but it is not necessary to show that any actual loss or gain occurs.

B) The defendant must intend or be reckless as to making a gain for himself or another or causing loss to another in property or money terms.

C) The defendant must intend to make a gain or cause loss in property or money terms and such a loss or gain must be proved to have occurred

A

A) The defendant must intend to make a gain or cause loss in property or money terms but it is not necessary to show that any actual loss or gain occurs

(Correct. The actus reus of fraud by false representation does not include a requirement of actual gain or loss. This is a conduct crime, committed as soon as the false representation is made)

127
Q

Which one of the following statements most accurately represents the mens rea of aggravated criminal damage?

A) Intention or recklessness as to destroying or damaging property and being reckless as to endangering life

B) Intention or recklessness as to destroying or damaging property and intention or recklessness as to thereby endangering the life of another

C) Intention or recklessness as to destroying or damaging property or intention or recklessness as to endangering the life of another

A

B) Intention or recklessness as to destroying or damaging property and intention or recklessness as to thereby endangering the life of another

128
Q

At what point in time must force be used to transform theft into robbery?

A) After the property has been taken into the defendant’s possession

B) At any time

C) Immediately before or at the time of the theft

D) It must always be used before the theft occurs

E) While the defendant is escaping having committed theft

A

C) Immediately before or at the time of the theft

(Correct. This is as stated in s 8(1) Theft Act 1968)

129
Q

Involuntary intoxication may be raised successfully as a defence to which crimes?

A) Crimes of both specific and basic intent involving dangerous or non-dangerous substances

B) Crimes involving dangerous drugs only

C) Crimes involving alcohol only

D) Crimes of basic intent only

E) Crimes of specific intent only

A

A) Crimes of both specific and basic intent involving dangerous or non-dangerous substances

(Correct. The key question is did the defendant still form the necessary mens rea? See R v Pordage)

130
Q

What is the question a court will ask regarding a defendant’s mens rea if they are voluntarily intoxicated by dangerous drugs/ alcohol and commit a basic intent offence?

A) Was the defendant incapable of forming the mens rea?

B) Would the defendant have formed the mens rea if sober?

C) Did the defendant still form the necessary mens rea even though intoxicated?

A

B) Would the defendant have formed the mens rea if sober?

(Correct. The defendant will be deemed reckless if they would have foresaw the risk of harm if sober, **Coley, McGhee and Harris. **The court will ask did the defendant form the mens rea even though intoxicated, in cases of:
• Involuntary intoxication (such as being drugged without consent);
• Voluntary intoxication by non-dangerous drugs (e.g. Hardie, the D who took Valium to calm his nerves); or
• Voluntary intoxication and D has committed a specific intent crime (e.g. murder).
The case of Pordage confirmed that the question at issue is not whether the defendant wasincapable of forming the mens rea, but whether, even if still capable, they did form it)

131
Q

When will intoxication prevent the defendant from using another defence?

A) When the defendant believed the victim consented to the accidental infliction of injury due to their intoxication

B) When the defendant honestly believed the owner would have consented to the damage due to voluntary intoxication

C) When a drunken mistake caused the defendant to use self-defence

D) When pleading loss of control

E) When pleading diminished responsibility

A

C) When a drunken mistake caused the defendant to use self-defence

(Correct. If a defendant makes a drunken mistake as to the need to use self-defence, they cannot rely on that mistake.
The other answers were incorrect:
- Where a statutory defence allows for an honest belief, D will be able to use the defence even if their belief is due to voluntary intoxication, see **Jaggard v Dickinson **on the lawful excuse defence for criminal damage.
- If the jury are satisfied that V consented to the accidental infliction of injury or D (even wrongly) believed that V consented (due to their intoxication), D may have a defence, Richardson & Irwin.
- Intoxication is no bar to a plea of loss of control or diminished responsibility)

132
Q

Which of the following best explains when consent is a defence to an offence against the person?

A) Consent is only available to assault and battery unless one of the exceptions apply

B) Consent is only available to assault and battery

C) Consent is not available to any offences against the person

D) Consent is available to all offences against the person

A

A) Consent is only available to assault and battery unless one of the exceptions apply

(Correct. The general rule is that consent will operate as a defence to assault and battery only, AG’s Reference (No 6 of 1980). The case of R v Meachen extended the rule to provide that the defence could still operate even where ABH or worse was caused provided the defendant only intended to commit a battery with the consent of the victim and did not see the risk of causing ABH)

133
Q

Which of the following best explains whether consent is available for assault and battery?

A) Consent is available if V consented and D honestly believed that V was consenting

B) Consent is available if D honestly believed that V was consenting

C) Consent is available if V consented

D) Consent is available if V consented or D honestly believed that V was consenting

A

D) Consent is available if V consented or D honestly believed that V was consenting

(Correct. If the defendant wrongly believed the victim consented, the defence could be available, R v Richardson and Irwin. Equally if the victim consented, even if the defendant did not know this, the defence could be available)

134
Q

Consent is only available to assault and battery unless one of the exceptions apply. Which of the following exceptions are not available if D intended or was reckless as to causing ABH or above?

A) Lawful correction of a child

B) Personal adornment

C) Sport

D) Sexual gratification/ accidental infliction of harm

E) Horseplay

A

A) Lawful correction of a child

(Correct. The Children Act 2004, s 58 outlines that the reasonable punishment defence cannot be relied on if it results in ABH or above for example. Consent can be available in certain circumstances for sport, horseplay, personal adornment and sexual gratification/ accidental infliction of harm)

135
Q

Which of the following cannot be used as a trigger for self-defence?

A) D’s mistaken but unreasonable belief they were under attack

B) D’s mistaken but reasonable belief they were under attack

C) D’s use of anticipatory self-defence in the belief they were under imminent attack

D) D’s mistaken belief they were under attack induced by voluntary intoxication

A

D) D’s mistaken belief they were under attack induced by voluntary intoxication

(Correct. Self-defence cannot be relied on if a mistake is induced by voluntary intoxication, R v O’Connor ands 76(5))

136
Q

Which of the following accurately describes the question to be asked when judging whether the response was reasonable for the purposes of self-defence?

A) Was the level of force used objectively reasonable in the circumstances as D subjectively believed them to be?

B) Was the level of force objectively reasonable?

C) Did D believe the level of force used to be reasonable in the circumstances?

A

A) Was the level of force used objectively reasonable in the circumstances as D subjectively believed them to be?

(Correct. This reflects the common law position and the s 76(3) Criminal Justice and Immigration Act 2008. The trigger aspect of the test has objective and subjective elements to it)

137
Q

Self-defence cannot be relied on which of the following circumstances?

A) To protect against psychological harm

B) To protect yourself against attack

C) To protect yourself, another or property against imminent attack

D) To protect another against attack

E) To protect property against attack

A

A) To protect against psychological harm

(Correct. Self-defence cannot be relied on in these circumstances, see R v Bullerton)

138
Q

Which of the following best describes when force will not be reasonable in householder cases?

A) If it was grossly disproportionate in the circumstances as D believed them to be

B) If it was grossly disproportionate

C) If it was disproportionate in the circumstances as D believed them to be

D) If it was disproportionate

A

A) If it was grossly disproportionate in the circumstances as D believed them to be

(Correct. If the force was grossly disproportionate, there can be no defence)

139
Q

Does the use of force in householder cases need to be reasonable?

A) No

B) Yes

C) Depends on the circumstances

A

B) Yes

(Correct. Confirmed by the High Court in the Denby Collins case and the Court of Appeal in **R v Ray (Steven). **If the force was not grossly disproportionate in the circumstances, the next question is whether the level of force was reasonable)

140
Q

To be potentially classed as a householder case, which of the following must the defendant rely on?

A) Preventing a crime

B) Assisting in the arrest of an offender

C) Protecting property

D) Protecting themselves or another

A

D) Protecting themselves or another

(Correct. This reflects the Criminal Justice and Immigration Act 2008, s 76(8A)(a))

141
Q

The general rule is that impossibility is not a defence to an attempted offence. Which one of the following types of impossibility might be said to act as a defence?

A) Impossibility through non-existent crimes

B) Impossibility through inadequacy

C) Impossibility in fact

A

A) Impossibility through non-existent crimes

(This is the correct answer. The Taaffe case demonstrates this legal principle. Impossibility through inadequacy and impossibility in fact are no defences to an attempted offence, section 1(2)&(3) Criminal Attempts Act 1981)

142
Q

What is the correct meaning of an inchoate offence?

A) Attempted

B) Conspired

C) Unfinished

A

C) Unfinished

(This is the correct answer. Inchoate offences are unfinished or incomplete offences)

143
Q

In which one of the following cases had the defendant done an act that was more than merely preparatory?

A) Jones

B) Geddes

C) Campbell

A

A) Jones

(This is the correct answer, it was more than merely preparatory to get into a car with a loaded gun and point it at the victim, therefore attempted murder. Being outside a post office with a threatening note and fake gun was held not to be attempted robbery, Campbell. Being in school toilets with a knife and rope but no schoolchildren)

144
Q

Which one of the following correctly describes the mens rea for attempted murder?

A) Intention to kill or cause serious harm

B) Intention to cause serious harm

C) Intention or recklessness as to killing

D) Intention to kill

A

D) Intention to kill

(This is the correct answer. See the case of Whybrow for further details. Intention to kill or cause serious harm is the mens rea of murder)

145
Q

The general rule is that impossibility is not a defence to an attempted offence. Which one of the following types of impossibility might be said to act as a defence?

A) Impossibility through non-existent crimes

B) Impossibility in fact

C) Impossibility through inadequacy

A

A) Impossibility through non-existent crimes

(This is the correct answer. The Taaffe case demonstrates this legal principle. Impossibility through inadequacy and impossibility in fact are no defences to an attempted offence, section 1(2)&(3) Criminal Attempts Act 1981)

146
Q

Donna is fed up with her husband Tim. She tells her sister Ursula that she wants to kill him. Ursula lends Donna a gun that Ursula bought the previous week. Donna produces the gun that night whilst Tim is in bed and pulls the trigger. Unknown to either Donna or Ursula, the gun is a replica which is unable to fire. Donna is charged with attempted murder.

Which of the three types of impossibility would be relevant here?

A) Impossibility through inadequacy

B) Non-existent crimes

C) Impossible in fact

A

A) Impossibility through inadequacy

(Correct. This argument would not succeed, it is immaterial that Donna has used an inadequate method to achieve her aim, section 1(2) & (3) Criminal Attempts Act 1981)

147
Q

Kevin breaks into a garage that his next door neighbour uses and steals a bicycle. When Kevin’s solicitor investigates the ownership of the garage, he finds that the neighbour has built the garage on land owned by Kevin. He has been charged with attempted burglary. Kevin has broken onto his own land.

Which of the three types of impossibility would be relevant here?

A) Impossibility through inadequacy

B) Impossible in fact

C) Non-existent crimes

A

B) Impossible in fact

(Correct. Kevin would seek to argue that there is no offence here as the result Kevin intends will not be the crime he thinks it will be. However this would fail as following s 1(2)&(3) Criminal Attempts Act 1981, impossibility is no bar to a charge of attempted burglary)

148
Q

What is the term for the person who commits an offence with the appropriate actus reus and mens rea?

A) Party to a joint enterprise

B) Principal

C) Accessory

A

B) Principal

(Correct. Broadly an accessory is someone that assists or encourages the commission of an offence, rather than commits it. While a party to a joint enterprise will be a joint principal, there needs to be more than one person for a joint enterprise)

149
Q

Which of the following cannot lead to criminal liability?

A) To aid, abet, counsel or procure an attempt to commit an offence

B) To aid, abet, counsel or procure an offence

C) To attempt to aid, abet, counsel or procure an offence

A

C) To attempt to aid, abet, counsel or procure an offence

(Correct. This is not an offence. Essentially, you cannot fulfil the actus reus of accessorial liability by an attempt.
To aid, abet, counsel or procure an offence- is an offence, these are some of the ways in which you can fulfil the actus reus of accessorial liability.
To aid, abet, counsel or procure an attempt to commit an offence- is an offence, these are some of the ways in which you can fulfil the actus reus of accessorial liability- here it just happens to be that the principal is committing an attempt)

150
Q

Who decides whether a person has withdrawn from a joint enterprise?

A) The other party to the joint enterprise

B) The jury

C) The defendant

A

B) The jury

(Correct. It is a question of fact and degree for the jury. Account will be taken of the nature of the assistance and encouragement already given and how imminent the infliction of the fatal injury or injuries is, as well as the nature of the action said to constitute withdrawal.
It is not the defendant as the general rule is that it is not enough to just have a change of mind. Something must be done and, at the very least, the withdrawal must be communicated to the principal or a law enforcement agency)

151
Q

The actus reus of accessorial liability can be committed in five ways. Which of the following requires a causal link between D’s act and what P does?

A) To aid

B) To counsel

C) To procure

D) To abet

E) To be a party to a joint enterprise

A

C) To procure

(Correct. There must be a causal link between D’s act and the commission of the offence)

152
Q

Which of the following is not part of the mens rea for accessorial liability?

A) An intention to assist or encourage the principal’s conduct

B) An intention that the principal will do the actus reus with that mens rea

C) An intention to do the actus reus and mens rea of the principal offence

D) Knowledge of existing facts or circumstances necessary for the offence to be criminal

A

C) An intention to do the actus reus and mens rea of the principal offence

(Correct- this would make D the principal rather than an accessory. The mens rea for accessorial liability is:
• An intention to assist or encourage the principal’s conduct.
• If the crime requires a mens rea, an intention that the principal will do the actus reus with that mens rea.
• Knowledge of existing facts or circumstances necessary for the offence to be criminal)

153
Q

The actus reus of accessorial liability can be committed in five ways. If D gives the principal advice or encouragement before the offence is committed, how would that be categorised?

A) To procure

B) To counsel

C) To aid

D) To be a party to a joint enterprise

E) To abet

A

B) To counsel

(Correct. There must be contact and consensus between P and D along with a connection between the advice and the crime. Causation isn’t needed.
To aid is giving help, support or assistance before or at the time of the offence e.g. giving information, supplying tools or driving P.
To abet is to incite, instigate or encourage P at the time of the offence which must be communicated to P.
To procure P to commit the offence is to produce by endeavour.
An example of a joint enterprise is where the P and D commit burglary as principals, P murders the homeowner, D becomes an accessory to P’s murder)

154
Q

A brother and sister have a fight, during which the brother is knocked unconscious. The sister runs off. Soon after, a gang member appears and stabs the brother in the chest. He dies from the stab wound.

Has the sister caused her brother’s death, as required for the actus reus of murder?

A) She has caused her brother’s death because but for the sister rendering her brother unconscious, the gang member would not have been able to stab him

B) She has not caused her brother’s death because the gang member’s act is a free, deliberate and informed act

C) She has not caused her brother’s death because the gang member’s action was not reasonably foreseeable

D) She has caused her brother’s death as she must take her brother as she finds him, with a gang member that wants to kill him

E) She has caused her brother’s death because there can be more than one cause

A

B) She has not caused her brother’s death because the gang member’s act is a free, deliberate and informed act

(Correct. To be the cause of her brother’s death she must be the factual and legal cause. This question is about legal causation and whether the sister is the operating cause of her brother’s death. The chain of causation is broken by the act of the gang member. It was held in R v Pagett that the chain of causation will be broken by a free, deliberate and informed act of a third party.
The other answers were incorrect.
The ‘reasonably foreseeable’ test is the one applied when considering whether acts of the victim will break the chain of causation.
The answers that stated the sister caused her brother’s death overlooked whether the gang member’s act broke the chain of causation:
· While the sister is the factual cause of her brother’s death, she must also be the legal cause.
· It is true that the sister’s act need not be the only cause of her brother’s death, it is enough that it is a substantial cause. However, the sister’s act must also be an operating cause.
· While the sister must take her brother as she finds him, the thin skull rule tends to be applied to issues such as a pre-existing infirmity or peculiarity)

155
Q

A man lives alone and is reliant upon the care provided to him by his niece following an injury at work he suffered a year ago. The niece has cared for the man throughout this year. This includes bringing the man food and collecting his medications from the pharmacy. The niece accepts money from her uncle for these items but does not accept his offer of payment for her time in doing this. The niece becomes very busy at work and does not call on her uncle as often as she used to, which means that the man only receives food irregularly and has been without medication for a week. One night the niece notices that her uncle has become unwell and assists him into bed to rest. The man dies in the night.

Which of the following best describes the niece’s liability for the man’s death?

A) The niece may be liable for the man’s death as she had a special relationship to him.

B) The niece may be liable for the man’s death as she had voluntarily assumed a duty of care towards him

C) The niece is not liable for the man’s death because there can be no liability for an omission

D) The niece is liable for the man’s death as she has a contractual duty to her uncle

E) The niece is liable for the man’s death as she has created a dangerous situation in leaving him without medical assistance

A

B) The niece may be liable for the man’s death as she had voluntarily assumed a duty of care towards him

(Correct. There is generally no duty to act to prevent harm, R v Smith (William). However, the niece has voluntarily assumed a duty of care towards the man by caring for him for the last year and therefore may be liable for his death, R v Stone and Dobinson. While the other options might sound plausible, they are each incorrect. A defendant can be liable for an omission if they have a legal duty to act. The relationship between an uncle and a niece is likely to not be sufficiently proximate for a duty of care to arise by virtue of the special relationship duty. Although the niece accepts money to reimburse her for the items she collects, she has refused her uncle’s payment for her time and so there is no indication of a contract of employment. Finally, this scenario is more analogous to the voluntary assumption of a duty of care cases, than creating a dangerous situation as illustrated by the case of Miller for example)

156
Q

A woman owns a coffee shop in her local town. A new coffee shop has been opened at the other end of the high street which is affecting the woman’s business. Late one night the woman goes to the new coffee shop with a can of petrol. A light shows from a first floor window above the coffee shop and she can see the shadow of a person in that light. However, she is determined to destroy the new coffee shop. She breaks a window, pours petrol through the broken window and then throws in a lighted rag. The man living in the flat above the coffee shop dies in the ensuing fire.

Which of the following question(s) will best assist the jury in determining whether the woman is liable for murder?

A) Was it virtually certain that someone would die or suffer grievous bodily harm as a result of the defendant’s act?

B) Was the woman aware of the risk that someone would die or suffer serious harm as a result of her actions and in the circumstances known to her, was it an unreasonable risk to take?

C) Was it virtually certain that someone would die or suffer serious harm as a result of the woman’s act and did the woman appreciate that?

D) Was it virtually certain that someone would die or suffer grievous bodily harm as a result of the defendant’s act and would a reasonable person appreciate that?

E) Was it the woman’s aim or purpose to kill or cause grievous bodily harm?

A

C) Was it virtually certain that someone would die or suffer serious harm as a result of the woman’s act and did the woman appreciate that?

(Correct. The woman’s direct intention, her aim and purpose, is to destroy the new coffee shop. The jury will be asked to consider oblique intent and this is the test for oblique intent from R v Woollin.
The other answers were incorrect as:
· Misstated the R v Woollin questions regarding oblique intention.
· Applied direct intention to kill or cause grievous bodily harm when this would not be the best question to help determine the woman’s liability for murder in this case.
· Applied the test of recklessness and you cannot commit murder recklessly)

157
Q

A woman crosses a red light on her bicycle and accidentally hits a pedestrian. The pedestrian’s neck is broken and it is doubtful whether he will live. The pedestrian claims damages from the woman. The woman’s solicitor tells her she was negligent and will be liable for several million pounds in damages if the pedestrian survives. If he dies she will have to pay a few thousand pounds as the pedestrian has no dependents. The woman is not covered by insurance so will lose her house and all her savings if the pedestrian survives. She is at an age where she will not be able to buy another house or save much for her retirement. She begins to formulate a plan to smother the pedestrian. Before she can take any action, the pedestrian dies as a result of the injuries sustained in the collision and the woman is pleased he has died.

Which of the following best describes the woman’s liability for murder?

A) She is guilty because at the time of the pedestrian’s death the woman had the mens rea of murder

B) She is not guilty because she does not have the mens rea of murder given she did not put her plan into action

C) She is not guilty because there is no coincidence of actus reus and mens rea

D) She is guilty because there is coincidence of actus reus and mens rea by virtue of this being a series of acts that form one transaction

E) She is guilty because there is coincidence of actus reus and mens rea by virtue of the continuing act theory

A

C) She is not guilty because there is no coincidence of actus reus and mens rea

(Correct. The actus reus took place at the time of the act (hitting the pedestrian with the bicycle) which ultimately caused death. The mens rea for murder is intention or kill or intention to cause grievous bodily harm. At the time of hitting the pedestrian, the woman did not have the mens rea of murder.
The other answers were incorrect. The continuing act theory cannot be used as the woman’s act of hitting the pedestrian had finished long before the woman intended the pedestrian’s death. The series of acts theory cannot be used as this is not a case where the woman initially acts with the mens rea and a later act designed to cover up the first act causes death, as in R v Thabo Meli. The woman had only done one act and she did not have the mens rea at that time. Formulating a plan to smother the pedestrian constituted the mens rea of murder, intention to kill, regardless of whether she took any steps to put her plan into action. The actus reus did not take place at the time of the pedestrian’s death)

158
Q

A woman owns a coffee shop in her local town. A new coffee shop has been opened at the other end of the high street which is affecting the woman’s business. Late one night the woman goes to the new coffee shop with a can of petrol. A light shows from a first floor window above the coffee shop and she can see the shadow of a person in that light. However, she is determined to destroy the new coffee shop. She breaks a window, pours petrol through the broken window and then throws in a lighted rag. The man living in the flat above the coffee shop dies in the ensuing fire.

Which of the following question(s) will best assist the jury in determining whether the woman is liable for murder?

A) Was it virtually certain that someone would die or suffer grievous bodily harm as a result of the defendant’s act?

B) Was the woman aware of the risk that someone would die or suffer serious harm as a result of her actions and in the circumstances known to her, was it an unreasonable risk to take?

C) Was it the woman’s aim or purpose to kill or cause grievous bodily harm?

D) Was it virtually certain that someone would die or suffer grievous bodily harm as a result of the defendant’s act and would a reasonable person appreciate that?

E) Was it virtually certain that someone would die or suffer serious harm as a result of the woman’s act and did the woman appreciate that?

A

E) Was it virtually certain that someone would die or suffer serious harm as a result of the woman’s act and did the woman appreciate that?

(Correct. The woman’s direct intention, her aim and purpose, is to destroy the new coffee shop. The jury will be asked to consider oblique intent and this is the test for oblique intent from R v Woollin.
The other answers were incorrect as:
· Misstated the R v Woollin questions regarding oblique intention.
· Applied direct intention to kill or cause grievous bodily harm when this would not be the best question to help determine the woman’s liability for murder in this case.
· Applied the test of recklessness and you cannot commit murder recklessly)

159
Q

A man spots a drugs dealer from whom he has stolen a large supply of drugs, in the High Street. Hoping the drugs dealer has not seen him, the man slips into an shop and runs up the stairs to the first floor. The drugs dealer has seen the man, so follows him into the shop and up the stairs. The drugs dealer approaches the man with a large knife, shouting ‘you will learn not to mess with me.’ Cornered, the man jumps out of the window. The man breaks both legs on the pavement below. The actus reus for the offence with which the drugs dealer is charged requires him to have caused serious harm.

Which of the following best explains how the jury will decide whether the drugs dealer has the actus reus for the offence?

A) If the jury decide that what the man did was daft, unexpected and unreasonable, then the drug dealer will not have caused the man serious harm

B) If the jury decide that what the man did was so daft and unexpected that no reasonable person could have foreseen it, then the drug dealer will not have caused the man serious harm

C) If the jury decide that the drug dealer intended to cause the man serious harm with his knife, then the drug dealer will have caused the man serious harm, even though it occurred in a different way than the drug dealer intended

D) If the jury decide that what the man did was free, deliberate and informed, then the drug dealer will not have caused the man serious harm

E) If the jury decide that what the man did was so independent of the drug dealer’s acts, and in itself so potent in causing serious harm, then the drug dealer will not have caused the man serious harm

A

B) If the jury decide that what the man did was so daft and unexpected that no reasonable person could have foreseen it, then the drug dealer will not have caused the man serious harm

(Correct. The man’s two broken legs constitute serious harm. In order to say that the drug dealer caused the man’s serious harm, he must be the factual and legal cause of that harm. The key issue here is one of legal causation- whether the drug dealer’s act is the operating cause of the man’s broken legs or whether the chain of causation has been broken by the man jumping out of the window (a ‘fright and flight’ case). The question to consider is from R v Roberts which was approved by the Court of Appeal in R v Williams and Davies. Was the victim’s act reasonably foreseeable or was it so daft and unexpected that no reasonable person could have foreseen it?
The other answers were incorrect because whether:
- ‘daft, unexpected and unreasonable’- this is not the precise test approved by the Court of Appeal.
- the drug dealer intended to cause serious harm is a mens rea rather than an actus reus issue.
- what the man did was so independent of the drug dealer’s acts, and in itself so potent in causing serious harm- this is the test applied when considering whether medical negligence has broken the chain of causation.
- what the man did was free, deliberate and informed- this is the test applied when considering whether the acts of a third party have broken the chain of causation)

160
Q

The defendant was a glue sniffer who was high on glue. The victim taunted the defendant. As well as other abuse, the victim told the defendant that he was a useless character who had wasted his life and his talents due to his addiction. This hurt, as the defendant knew it was true. As a result of this abuse and the other taunts, the defendant lost his self-control and killed the victim.

How will the defendant’s addiction to glue and the fact that he is high on glue affect a defence of loss of control?

A) The defendant’s addiction to glue will be taken into account as a qualifying trigger and will be a characteristic given to the normal man. However, the normal man will not be high on glue

B) The defendant’s addiction to glue and the fact he is high on glue will preclude him from using the defence of loss of control

C) The defendant’s addiction to glue and the fact he is high on glue will not be a characteristic given to the normal man, as this reduces his self-restraint

D) The defendant’s addition to glue will be taken into account as a qualifying trigger but will not be a characteristic given to the normal man

E) The defendant’s addiction to glue will not be taken into account at all but he can still use the defence of loss of control

A

A) The defendant’s addiction to glue will be taken into account as a qualifying trigger and will be a characteristic given to the normal man. However, the normal man will not be high on glue

(Correct. This reflects the position in R v Asmelash. The defendant’s addition to glue and the effect it has had on his life will be taken into account as a qualifying trigger under s 54(1)(b) Coroners and Justice Act 2009. It will also be a characteristic given to the normal man when the jury considers s 54(1)(c). However, the normal man will have normal levels of tolerance and self-restraint and will not be high on glue.
The other answers may have sounded plausible but were incorrect)

161
Q

A man has an argument with his partner and punches her in the shoulder. She falls over and hits her head on the edge of a table, sustaining head injuries from which she dies. He is charged with murder. He tells you that he had no intention to seriously injure her.

How would you advise the man on his criminal liability for murder?

A) He is not criminally liable for murder on the basis that he was acting in self-defence

B) He is not criminally liable for murder on the basis that he has the defence of diminished responsibility

C) He is criminally liable for murder on the basis that, by his own account, he does have the mens rea

D) He is not criminally liable for murder on the basis that, by his own account, he does not have the mens rea.

E) He is not criminally liable for murder on the basis that he has the defence of loss of control

A

D) He is not criminally liable for murder on the basis that, by his own account, he does not have the mens rea

(Correct. The mens rea for murder is an intention to kill or cause serious harm, R v Vickers. The man says he did not have either of these states of mind so he should plead not guilty to murder.
The other options were incorrect because:
· Punching his partner in the shoulder will not equate to serious harm.
· There is nothing in the facts to suggest, for example, that the man:
- could rely on the fear or anger qualifying trigger for the purposes of loss of control.
- was acting to protect himself, honestly believing that force was necessary for the purposes of self-defence.
- was suffering from a recognised medical condition for the purposes of diminished responsibility)

162
Q

A man spends the evening drinking a lot of alcohol with friends. At the end of the evening, while he is waiting for a taxi, a woman pushes in front of him in the queue. Furious, he deliberately pushes her in front of a passing car which crashes into her. She dies of her injuries.

Which of the following best describes why the defence of diminished responsibility is not available to the man?

A) Voluntary intoxication does not provide an explanation for the man’s actions

B) It was necessary for the man to have been involuntarily intoxicated

C) Voluntary intoxication doesn’t substantially impair the man’s ability to exercise self-control

D) Voluntary intoxication is not, on its own, capable of being relied upon to found this defence

E) It was necessary for the man to have been suffering from an inability to resist the urge to drink

A

D) Voluntary intoxication is not, on its own, capable of being relied upon to found this defence

(Correct. We are told that the man has spent the evening drinking a lot of alcohol. The case of Dowds makes it clear that this, on its own cannot give rise to a defence of diminished responsibility.
The other options were incorrect because:
· Section 2(1B) Homicide Act 1957 states that an abnormality of mental functioning provides an explanation for D’s conduct if it causes or is a significant contributory factor in causing, D to carry out that conduct. The high amount of alcohol the man drank with friends is likely to have caused or been a significant contributory factor in causing him to push the woman in front of the passing car.
· It is likely that the man’s voluntary intoxication was the reason why he was not able to exercise self-control when the woman pushed in front of him in the queue. However, in order to rely on diminished responsibility, the man needs to satisfy all of the requirements in section 2(1) Homicide Act 1957, including an abnormality of mental functioning which arose from a recognised medical condition. There is no recognised medical condition on the facts.
· It is possible for the man to rely on the defence of diminished responsibility, even he was not suffering from alcohol dependency syndrome, R v Dietschmann.
· Being voluntarily intoxicated does not preclude the man from using the defence. A defendant might, at the time of the killing suffer from both an abnormality of mental functioning and from the effect of alcohol taken before the killing, R v Dietschmann)

163
Q

A woman loses her temper very easily. She has been going to anger management classes in an attempt to overcome this failing, but they have not been working. One day she returns from work to find her boyfriend in a passionate embrace with her 15-year-old daughter. She picks up a heavy doorstop and hits him repeatedly on the head. He dies. She is pleading the defence of loss of control.

What effect would the woman’s short temper have on the requirement that she must have killed as a result of a loss of self-control?

A) She can use the defence of loss of control as a loss of temper is only considered at the third requirement, the normal person test

B) She cannot use the defence of loss of control as she does not have a normal degree of tolerance and self-restraint

C) She cannot use the defence of loss of control as she will be unable to prove beyond reasonable doubt that she did not act out of a loss of temper

D) She can use the defence of loss of control as a loss of temper is a loss of self-control

E) She can use the defence of loss of control if she killed as a result of more than a loss of temper but loss of self-control

A

E) She can use the defence of loss of control if she killed as a result of more than a loss of temper but loss of self-control

(Correct. This answer reflects the position in R v Richens.
The other answers are incorrect because:
· A loss of temper is not the same as a loss of self-control.
· The third requirement of the loss of control defence is that a person of D’s sex and age, with a normal degree of tolerance and self-restraint and in the circumstances of D, might have reacted in the same or a similar way to D. This means that the woman’s short temper will not be a characteristic given to the normal person, but it does not preclude her from using the defence of loss of control.
· The prosecution has the burden of proof to prove the woman acted from a loss of temper rather than a loss of self-control, s 54(5) Coroners and Justice Act 2009.
· Factors such as a loss of temper which go to the defendant’s capacity for tolerance and self-restraint are considered in the third requirement of the defence, where they may be excluded if they have not formed part of the qualifying trigger. However, loss of temper will be considered in the first requirement of the defence, that she must have killed as a result of a loss of self-control)

164
Q

A man lives in an area where there has been a spate of burglaries in which violence has been used against the occupiers. He is very afraid of being the next victim, so he acquires a shotgun which he keeps by his bed. Early one morning, he hears the sound of breaking glass coming from downstairs. He picks up the shotgun and makes his way quietly downstairs. He sees that the window in the front room has been broken and a shadowy figure is moving about. He points the shotgun at the figure who, seeing him, dashes to the window and begins to climb out. Unable to restrain himself, the man shoots and hits the figure in the back, killing him instantly. The man is charged with murder.

Why can’t the man rely on the defence of loss of control?

A) The man acted in a considered desire for revenge

B) The man didn’t lose self-control

C) The man has not lost control as a result of a qualifying trigger

D) The man acted in self-defence

E) The man fails the normal person test

A

E) The man fails the normal person test

(Correct. A person with a normal degree of tolerance and self-restraint in his circumstances would not have reacted in the same or a similar way, s 54(1)(c). As the victim is clearly intent on running away, a person with a normal degree of tolerance and self-restraint in his circumstances would not have shot him in the back.
The other answers were incorrect because:
· The facts state that the man was ‘unable to restrain himself’ which is a loss of self-control according to R v Richens.
· While acting in a considered desire for revenge is a reason why loss of control is not available (s 54(4)), there is no suggestion of this on the facts. Even if the man shot the person in the back to take revenge for breaking the window and entering his house, it was not ‘considered.’
· The man could potentially rely on both the ‘fear’ and the ‘anger’ triggers (sections 55(3) and 55(4) Coroners and Justice Act 2009). The man fears serious violence given the spate of violent burglaries. The man also reacted to a thing done (the broken window and figure entering his house) which could be argued to constitute circumstances of an extremely grave character which caused him to have a justifiable sense of being seriously wronged.
· In order to rely on self-defence, one of the requirements is that the level of force the man uses must be reasonable and in the case of ‘householders’, reasonable means not grossly disproportionate, by reference to the circumstances as the defendant believed them to be. A fatal force to the person leaving the man’s house is unlikely to be considered a reasonable level of force. See s 76 Criminal Justice and Immigration Act 2008)

165
Q

The defendant was a glue sniffer who was high on glue. The victim taunted the defendant. As well as other abuse, the victim told the defendant that he was a useless character who had wasted his life and his talents due to his addiction. This hurt, as the defendant knew it was true. As a result of this abuse and the other taunts, the defendant lost his self-control and killed the victim.

How will the defendant’s addiction to glue and the fact that he is high on glue affect a defence of loss of control?

A) The defendant’s addiction to glue will not be taken into account at all but he can still use the defence of loss of control

B) The defendant’s addiction to glue and the fact he is high on glue will not be a characteristic given to the normal man, as this reduces his self-restraint

C) The defendant’s addiction to glue will be taken into account as a qualifying trigger and will be a characteristic given to the normal man. However, the normal man will not be high on glue

D) The defendant’s addition to glue will be taken into account as a qualifying trigger but will not be a characteristic given to the normal man.

E) The defendant’s addiction to glue and the fact he is high on glue will preclude him from using the defence of loss of control

A

C) The defendant’s addiction to glue will be taken into account as a qualifying trigger and will be a characteristic given to the normal man. However, the normal man will not be high on glue

(Correct. This reflects the position in R v Asmelash. The defendant’s addition to glue and the effect it has had on his life will be taken into account as a qualifying trigger under s 54(1)(b) Coroners and Justice Act 2009. It will also be a characteristic given to the normal man when the jury considers s 54(1)(c). However, the normal man will have normal levels of tolerance and self-restraint and will not be high on glue.
The other answers may have sounded plausible but were incorrect)

166
Q

A woman owns a coffee shop in her local town. A new coffee shop has opened at the other end of the high street which is affecting the woman’s business. Late one night the woman goes to the new coffee shop with a can of petrol. A light is on in a first-floor window above the shop and she can see the shadow of a person in that light. However, she is determined to destroy the new coffee shop. She breaks a window, pours petrol through the broken window and then throws in a lighted rag. The man living in the flat above the coffee shop escapes down an external fire escape at the back of the building. The woman is being charged with aggravated arson.

Which of the following questions are most likely to result in the woman being found to have the endangering life element of the mens rea for aggravated arson?

A) Did the woman see that it was virtually certain that life would be endangered by setting fire to the shop?

B) Was it the woman’s aim or purpose to endanger life by setting fire to the shop?

C) Did the woman see the risk of endangering life by setting fire to the shop and was the risk she saw an unreasonable one to take?

D) Was it virtually certain that life would be endangered by setting fire to the shop and did the woman see that it was virtually certain?

E) Did the woman see the risk of endangering life by setting fire to the shop and was the risk she saw an unreasonable one to take in the circumstances known to her, bearing in mind, that there is no social utility in the woman’s actions

A

E) Did the woman see the risk of endangering life by setting fire to the shop and was the risk she saw an unreasonable one to take in the circumstances known to her, bearing in mind, that there is no social utility in the woman’s actions

(Correct. This is the test from R v G.
The other options, while plausible are incorrect or not the best answer.
The test in R v G is a two-part test. The second part is objective and requires a consideration of whether the risk seen by the defendant was, objectively, an unreasonable one to take in the circumstances known to the defendant.
Where the mens rea of an offence is defined as intention or reckless, if direct intention cannot be established, oblique intention should not be applied. Instead, the test for recklessness should be applied.
The woman’s direct intention is to destroy the new coffee shop, not to endanger the life of the man living in the flat above. The woman is more likely to be found to have the endangering life element of the mens rea for aggravated arson by applying the test for recklessness as set out in R v G)

167
Q

A woman tips some cooking oil from her second floor flat onto the pavement below to avoid blocking the drains with fat. A man slips on the oil and breaks his hip.

What is test for the woman to be reckless as to causing physical harm?

A) The risk of causing physical harm which she took must be an unreasonable one when weighed against the social utility in not blocking drains with fat

B) She must have seen the risk of causing physical harm and went ahead anyway

C) She must have given no thought to the risk of causing physical harm and the risk must be an unreasonable one to take

D) She must have seen the risk of causing physical harm and a reasonable person must consider the risk an unreasonable one to take

E) She must have seen the risk of causing physical harm and the risk she saw must be an unreasonable one to take

A

E) She must have seen the risk of causing physical harm and the risk she saw must be an unreasonable one to take

(Correct. This is the test from R v G: a defendant is reckless with respect to a result when aware of a risk that it will occur and it is, in all the circumstances known to the defendant, unreasonable to take the risk. The first issue is judged subjectively; the second, objectively.
The other options, while plausible are incorrect, often because they are not accurate enough.
The question of whether the risk is an unreasonable one to take, is judged objectively in the light of the circumstances known to the defendant.
Both elements of the test must be satisfied. So, the defendant must see the risk and it must be an unreasonable one to take in the circumstances known to her.
If the defendant has given no thought to the possibility of the risk, then she does not see the risk and so is not reckless as one element of the test is missing.
The fact that the defendant sees the risk and goes ahead anyway is not evidence of recklessness. The question is whether the risk recognised by the defendant is an objectively unreasonable one to take in the circumstances known to the defendant)

168
Q

A man puts a lighted rag through a letter box. He wants to burn the house down. The flames destroy a rug and damages a chair and some coats in the hall, but all the doors from the hall are shut and the fire eventually burns itself out on the stone floor beneath the rug.

Is the man criminally liable for aggravated arson?

A) The man will be liable if he intended or it was virtually certain that life would be endangered by burning the house down

B) The man will be liable because when damage is caused by fire, there is always a risk to life from the damaged property

C) The man will be liable because even if he did not intend to endanger life by the damaged property, there was an obvious risk of endangering life by burning the house down

D) The man will not be liable as no life was endangered

E) The man will be liable if he intended or was reckless as to endangering life by burning the house down

A

E) The man will be liable if he intended or was reckless as to endangering life by burning the house down

(Correct. See R v Dudley. The words destruction and damage in s 1(2) Criminal Damage Act 1971 refer to the destruction and damage intentionally or recklessly caused (burning down the house), not the destruction and damage which actually occurred (damage done to the rug, chair and coats). The man’s mens rea as to endangering life will be established by reference to the damage he intended or saw a risk of causing.
The other options are incorrect.
For aggravated criminal damage or aggravated arson, no life need be endangered in fact, R v Sangha.
If the man did not directly intend to endanger life, recklessness should be considered as an alternative rather than oblique intention.
What Lord Bridge said in R v Steer, is that in cases where D intended or saw a risk of endangering life by fire it will be from damaged property. He did not say that whenever D damaged property by fire there would be a risk of endangering life.
Where it is alleged that the defendant was reckless as to endangering life by the damage, the test as set out in R v G will be applied. Consider first whether the defendant did see the risk. Next, apply the objective test – was the risk seen by the defendant a reasonable one to take in the circumstances known to the defendant?)

169
Q

A man lives in a town with two rival football teams, the Town and Rangers teams. He and most of the people in his road are avid supporters of the Town team and live near the Town stadium, where the two teams are playing each other on Saturday. On Friday night the man gets very drunk and sprays, in red paint, on his neighbour’s house, ‘If any Rangers supporters come near here, we will break their legs!’ The man’s neighbour supports the Town team and the man thinks his neighbour will be pleased. In fact, the neighbour is furious.

If the man is charged with simple criminal damage, can he rely on the lawful excuse defence of belief in the owner’s consent?

A) Yes- the man believes the neighbour will consent to the damage to the property

B) No- the man cannot use the defence as he is intoxicated

C) Yes- the reasonable person would believe the neighbour will consent to the damage

D) No- the man cannot use the defence because the neighbour does not consent

E) No- the man cannot use the defence as his belief is not a reasonable one

A

A) Yes- the man believes the neighbour will consent to the damage to the property

(Correct. The defence of lawful excuse is set out in s 5 Criminal Damage Act 1971 and s 5(2)(a) is relevant here: the defendant acts in the belief that the owner consents to the damage or would do so if he knew of the damage and its circumstances. It is immaterial whether the belief is justified or not, so long as it is honestly held, s 5(3), and it does not matter that the man makes a mistake due to intoxication, see Jaggard v Dickinson.
The other options are incorrect.
The test is applied subjectively and if the defendant honestly believes that they have the owner’s consent to cause damage, the defence will apply even when that belief arises out of a drunken mistake.
The defence will also apply if the defendant honestly believes he has the owner’s consent for the damage.
The belief in consent does not have to be a reasonable one, so long as it is honest.
The defence is based on the defendant’s belief in consent. It is immaterial whether the owner does or does not consent to the damage in fact)

170
Q

A farmer is on his farm with a shotgun for shooting vermin. Suddenly he is passed by a car being driven haphazardly by a child. He recognises the car as belonging to his tractor driver. The car is heading straight for the farmer’s own vehicle. The farmer raises his shotgun and shoots out two of the cars’ tyres. The farmer is charged with aggravated damage.

Which of the following best explains the application of the lawful excuse defences?

A) The farmer cannot use the lawful excuses defences for an aggravated damage offence

B) The farmer can only use the lawful excuse of belief in the owner’s consent as he has not used reasonable force to protect his car from immediate harm

C) The farmer cannot use the lawful excuse defences as he has neither the owner’s consent nor does he act reasonably to protect his own property

D) The farmer can use both lawful excuse defences as the reasonable person would believe in the owner’s consent and that his car is in immediate need of protection

E) The farmer can use both lawful excuse defences as he genuinely believes the tractor driver would consent and that his car is in immediate need of protection

A

A) The farmer cannot use the lawful excuses defences for an aggravated damage offence

(Correct. Section 5(1) Criminal Damage Act 1971 states that the lawful excuse defences in s 5(2) do not apply to aggravated criminal damage or aggravated arson.
The other options, some of which are plausible if this was a case of simple criminal damage, are incorrect.
Both s 5(2)(a) and (b) are based on the defendant’s genuine beliefs, either in the owner’s consent or that the property to be protected is in immediate need of protection and that the means adopted by the defendant are reasonable. The fourth requirement of the defence under s 5(2)(b), introduced in case law such as R v Hunt, asks whether the steps taken by the defendant are capable of protecting the property. It does not require the steps to be reasonable)

171
Q

A father buys three soft scoop ice creams from a van for himself and his children. He hands over £10. The man in the van mistakenly thinks the father has handed over a £20 note so gives him £12.50 change in coins. Initially the father does not notice he has too much change as he is grappling with the ice creams and the coins. Eventually the father sees he has been given too much money and decides to keep it. By this time, his children have eaten most of their ice cream.

Which of the following best explains whether the money belongs to another for the purposes of theft?

A) The money belongs to another as the father is under an obligation to deal with the money in a particular way

B) The money belongs to another because the father is under a legal obligation to restore the extra change which belongs to the ice cream seller alone

C) The money does not belong to another because it belongs to the father who has possession and control of it

D) The money belongs to another because the father is under an automatic obligation to restore property acquired by mistake

E) The money belongs to another because the father is under a legal obligation to restore the extra change which belongs to the ice cream seller and the father

A

E) The money belongs to another because the father is under a legal obligation to restore the extra change which belongs to the ice cream seller and the father

(Correct. Section 5(4) Theft Act 1968 states that where property is given to another by a mistake and there is a legal obligation to restore it, that property belongs to the other for the purposes of the Theft Act.
The other options, some of which are plausible, are not the best answer or are incorrect.
Where property has been acquired by a mistake, legal ownership passes to the recipient (the father). However, s 5(4) Theft Act 1968 provides that for the purposes of theft, the property will be regarded as still belonging to the original owner, the ice cream seller (as well as the recipient) if the recipient is under a legal duty to restore the property in whole or part.
Section 5(4) does not create an automatic legal obligation to restore. The section will operate only if such an obligation can be established in law such as Attorney-General’s Reference (No 1 of 1983).
Section 5(3) would only operate if the father was under a legal obligation to use the money in a particular way for the ice cream seller)

172
Q

A woman suspects her colleague at work is suffering from the early stages of dementia. She tells him she has got into debt and is due to have her electricity cut off. This is not true. She asks the colleague for £2,000. Although he does not believe her, he hands her the money in cash. He does this because he knows she is a single parent who is struggling financially, he has plenty of money and he feels sorry for her.

Which of the following best explains whether the woman has appropriated the money for the purposes of theft?

A) She has not appropriated the money because the man consented- he knew she was lying when he gave her the money

B) She has not appropriated the money because the man intended it to be a gift to her

C) She has appropriated the money because she thinks he has dementia and is taking advantage of him

D) She has appropriated the money because she has lied to him in her reasons for needing the money

E) She has appropriated the money despite his consent, and despite his intention to give her the money as a gift

A

E) She has appropriated the money despite his consent, and despite his intention to give her the money as a gift

(Correct. It was held in R v Gomez that you can appropriate with the owner’s consent as appropriation is an objective description of the act done independent of the mental state of either the owner or the defendant. Following on from this reasoning, the House of Lords said in R v Hinks that it is possible to appropriate a gift.
The other options are incorrect. When considering whether there has been an appropriation for the purposes of theft, it is irrelevant that:
- she has lied to him in her reasons for needing the money;
- the man intended the money to be a gift to her;
- the man consented- he knew she was lying when he gave her the money;
- she thinks he has dementia and is taking advantage of him)

173
Q

A woman goes to a party. All the coats are left upstairs in a bedroom. When she goes to collect her coat she sees a mink jacket. Horrified at the cruelty of a coat made out of animal fur, she takes a pair of scissors from the dressing table and cuts the jacket into small pieces which could not be sown together again. She leaves the pieces on the bed and goes home.

Which of the following best describes whether the woman has intention to permanently deprive for the purposes of theft?

A) The woman does not have intention to permanently deprive because she has not risked the jacket’s loss

B) The woman has intention to permanently deprive because she has rendered the jacket useless

C) The woman does not have intention to permanently deprive because she does not offer the jacket by way of sale, bargain with it or sell it

D) The woman does not have intention to permanently deprive because she has not borrowed the jacket in circumstances making it equivalent to an outright taking

E) The woman has intention to permanently deprive because she has taken away all of the jacket’s goodness, virtue and practical value

A

B) The woman has intention to permanently deprive because she has rendered the jacket useless

(Correct. The extended meaning in s 6(1) Theft Act 1968 is ‘to treat the thing as [her] own to dispose of regardless of the other’s rights.’ DPP v J held that this will happen if the jacket is rendered useless.
The other options, some of which are plausible, are not the best answers or are incorrect.
Borrowing property that belongs to another can amount to intending to treat it as one’s own to dispose of regardless of the other’s rights if it is for a period and in circumstances making it equivalent to an outright taking, s 6(1). However, the woman’s behaviour is not easily seen as borrowing.
Intending to treat the property in a manner which risks its loss may be evidence of intention to permanently deprive according to s 6(1) and case law such as R v Fernandes and R v Marshall. However, on the facts, the jacket has been rendered useless, as it cannot be sown together again)

174
Q

A man asks the receptionist at the front desk of a hotel to get him a taxi to the station. The receptionist rings for a taxi from the company which the hotel always uses. The taxi takes the man to the station. The man pays in cash and the taxi driver gives him change but accidentally gives the man an extra £50 note which had become stuck to the £10 note he intended to give the man. The man does not notice this until after the taxi has gone. The man considers how he could contact the taxi driver to return the note but concludes that this would be extremely difficult and that the chances of tracing the taxi driver are slight. He keeps the £50 note.

Which of the following best explains whether the man is dishonest for the purposes of theft?

A) The man is not dishonest as he believes that the taxi driver cannot be discovered by taking reasonable steps

B) The man may be dishonest as he might be found dishonest by the standards of ordinary decent people

C) The man may be dishonest as he did not take any reasonable steps to find the taxi driver

D) The man is not dishonest as ordinary decent people would not find him to be dishonest

E) The man may be dishonest as his belief that the taxi driver cannot be discovered by taking reasonable steps was not reasonable

A

A) The man is not dishonest as he believes that the taxi driver cannot be discovered by taking reasonable steps

(Correct. The exception to dishonesty as set out in s 2(1)(c) Theft Act 1968 will apply here. The man does not believe the owner can be found by taking reasonable steps. His belief does not need to be reasonable, R v Robinson.
The other options, some of which are plausible, are not the best answer or are incorrect.
Section 2(1)(c) applies where the defendant believes that the owner cannot be discovered on taking reasonable steps. It is not necessary to show that the defendant has taken any steps to find the owner in fact.
The test in s 2(1)(c) is a subjective one, based on the defendant’s beliefs.
If any of the exceptions set out in s 2(1) apply to the defendant, he is not dishonest and therefore the test in Ivey v Genting Casinos will not be applied)

175
Q

A woman is meeting her personal trainer after work. He likes to be paid in cash. She realises that she does not have enough cash so borrows £20 from the envelope in her desk drawer which contains contributions from her colleagues for a leaving present for their boss. She intends to replace the money.

Which of the following is the best argument for why she will escape criminal liability for theft?

A) She is not dishonest

B) She does not appropriate the money as she intends to replace it

C) The money does not belong to another as ownership of the money passed to the woman when she collected it

D) She does not have an intention to permanently deprive because she will return the money with all its goodness, virtue and practical value

E) She does not take property as money is not mentioned as a form of property

A

A) She is not dishonest

(Correct. She may believe her colleagues would consent to her borrowing the money, s 2(1)(a). In cases where any of the exceptions to dishonesty apply, the common law test in **Ivey v Genting Casinos **will not be considered. However, it is unlikely she would be viewed as dishonest under the Ivey test in any event.
If any element of the offence is missing, as here with dishonesty, the offence cannot be established.
The other options, some of which are plausible, are not the best answer or are incorrect.
All the other elements of theft can be established here.
She appropriates the money when she takes it from the envelope as this is an assumption of a right of the owner to handle it and spend it, s 3(1) and R v Morris.
Money is specifically listed as property in s 4(1).
The money belongs to another assection 5(3) will almost certainly apply here. It looks as if there is a legal obligation to spend the money in a particular way. See Davidge v Bunnett.
There is intention to permanently deprive as she does not intend to return the exact same note. See R v Velumyl)

176
Q

A man goes to his neighbour’s house, at his neighbour’s request, to feed the cats while the neighbour is on holiday. The man decides, before entering the house, that he will see if there is any money lying around which he can take. There is no money lying around.

Will the man be criminally liable for burglary on entering the house?

A) The man will be liable for burglary on entering the house if there is money in the house

B) The man will not be liable for burglary on entering the house as he believes he had the neighbour’s permission to be there

C) The man will not be liable burglary on entering the house as he does not find any money to steal

D) The man will not be liable for burglary on entering the house, but will be liable if, later, he enters a part of the house where he does not have his neighbour’s permission to be

E) The man will be liable for burglary on entering the house

A

E) The man will be liable for burglary on entering the house

(Correct. He will be liable for a s 9(1)(a) offence as he has entered the house as a trespasser. He knew he was a trespasser and, at the time of entry, intended to steal. See R v Jones and Smith for entry in excess of permission constituting trespass and AG’s Ref (Nos 1 & 2 of 1979) for conditional intention being equivalent to intention.
The other options are incorrect.
Trespass occurs where the defendant enters a building or part of a building either without any permission or in excess of permission to be there.
The defendant must know or be reckless as to the facts which make him a trespasser.
A conditional intention to steal will suffice as part of the mens rea for burglary.
For a s 9(1)(a) offence, it is not necessary to prove that theft occurs)

177
Q

A woman is walking her dog in the park. A man comes up to her and says quietly, so as not to attract attention from others in the park, ‘give me your handbag or I will kick your dog.’ The woman gives the man her handbag. The man kicks the dog anyway.

Has the man committed robbery?

A) Yes. The man has stolen and used force on the dog

B) No. The man has not used force on any person

C) No. The man has threatened force, but not against the person from whom he has stolen

D) No. The man has stolen and used force, but he did not use the force immediately before or at the time of stealing and in order to steal

E) Yes. The man has stolen, threatened force on the dog and done so immediately before stealing and in order to steal

A

B) No. The man has not used force on any person

(Correct. He has not used force on any person or put or sought to put any person in fear of being then and there subjected to force as the threat is directed at the dog.
The other options are incorrect.
‘Force’ can be satisfied in three ways; actual use of force, putting someone in fear that they will be then and there subjected to force, or seeking to put someone in fear that they will be then and there subjected to force.
The force used or threatened does not have to be directed towards the person from whom the property is stolen.
The force used or threatened must be against a person, not property (and a dog is considered property for these purposes).
Appropriation may be a continuous act and it is for the jury to decide whether it has come to an end. If force is used or threatened during the continuation of the appropriation, the defendant may be liable for robbery, R v Hale)

178
Q

A woman goes into the local shop and tells the shopkeeper, who is behind the counter, that she knows the house where the shopkeeper’s mother lives, several villages away. The woman says that unless the shopkeeper gives her five packets of cigarettes, she will break the mother’s leg. The shopkeeper is on the telephone to his mother who hears the woman’s threat and is terrified. The shopkeeper gives the woman the cigarettes.

Has the woman committed robbery?

A) The woman has not committed robbery because the shopkeeper agrees to give her the cigarettes

B) The woman has not committed robbery if she does not know that the shopkeeper’s mother is terrified

C) The woman has committed robbery as the shopkeeper is put in fear then and there that his mother will be subjected to force

D) The woman has not committed robbery because any force the shopkeeper’s mother might fear will be in the future

E) The woman has committed robbery if the shopkeeper’s mother fears she will carry out her threat

A

D) The woman has not committed robbery because any force the shopkeeper’s mother might fear will be in the future

(Correct. This answer reflects the wording of s 8(1) Theft Act 1968 and the requirement that a person is put in fear of being then and there subjected to force. The intended victim of the force (here the shopkeeper’s mother) is not present and lives several villages away, so any force would be in the future.
The other options are incorrect.
In the absence of a use of force, it must be shown that someone fears that they will be subjected to force there and then or that the defendant seeks to make them fear this. Sometime in the future is not enough.
The full offence of theft must be established before considering the other elements of the robbery offence. Appropriation can occur even where the owner consents to it, see R v Gomez.
It is not enough that someone fears that force will be used on another person. The person threatened must fear that they will be subjected to force then and there)

179
Q

A man goes to sit behind a lady aged 85 on a bus. The lady has put her handbag on the seat beside her. The man leans forward and says to her, ‘if you don’t let me take your handbag, you will get punched!’ The lady is hearing impaired so does not hear what the man has said. The man realises she does not hear. When the bus approaches a stop, he grabs the handbag and jumps off the bus.

Has the man committed robbery?

A) The man has committed robbery because force can be applied to the handbag as well as to the person

B) The man may have committed robbery if the jury decides the appropriation is continuing

C) The man has not committed robbery because the force element of robbery is missing

D) The man has committed robbery.

E) The man has not committed robbery because he has not used force in order to steal

A

D) The man has committed robbery

(Correct. He has sought to put the lady in fear of being then and there subjected to force. He has done this before stealing and in order to steal. He steals the bag when he grabs it.
The other options are incorrect.
‘Force’ can be established in three ways. Actual use of force is only one of them.
It is not necessary to show that there is a causal link between the force and the theft to establish the actus reus of robbery. The requirement for a causal link in the mens rea is satisfied here– he has used force in order to steal.
It is for the jury to decide when an appropriation has come to an end. However, where force is used or threatened before the appropriation takes place, it is unnecessary to apply the continuing act principle from R v Hale.
For robbery, the force must be used or threatened against a person, not property, R v Clouden)

180
Q

A boy cuts the shoulder strap of a woman’s bag and the bag falls to the ground. The boy picks it up and runs off with it. He goes around a corner, looks in the bag and can find nothing he considers worth taking. He leaves the bag on the ground. The woman felt nothing and didn’t notice that her bag was missing until a few minutes later. The bag is later returned to the woman.

Has the boy committed robbery?

A) The boy has not committed robbery as he has not appropriated property

B) The boy has not committed robbery as a jury would not find the boy dishonest in these circumstances

C) The boy has not committed robbery because he had no intention to permanently deprive the woman of the bag

D) The boy has committed robbery as he has used force on the woman through an object

E) The boy has not committed robbery as he has not used force against a person

A

E) The boy has not committed robbery as he has not used force against a person.

(Correct. The woman did not feel the indirect contact so this will not be enough to constitute force, see P and others v DPP.
The other options are incorrect.
While force does not require violence, force used against property must cause force against the person for robbery to apply.
Force to detach property may count as force on the person. However, in cases of indirect contact, if it is very minimal, it will not be enough for robbery to apply.
The full offence of theft must be established before considering whether it becomes robbery. The boy has appropriated property when he cut the shoulder strap of the woman’s bag, as he assumes a right of the owner, s 3(1) Theft Act 1968 and R v Morris. The boy is clearly dishonest here and the jury would find him so even if they went on to apply the Ivey v Genting Casinos test. A conditional intention to permanently deprive will suffice for the purposes of theft, AG’s Ref (Nos 1&2 of 1979))

181
Q

An art dealer displays a painting depicting Hong Kong harbour in the 19th century. It is marked ‘artist unknown.’ The art dealer does not know the name of the artist, but the third party from whom he bought the picture said it was painted by a friend of his. A buyer comes into the gallery and expresses interest in the painting. The art dealer says he believes the artist to be a famous 19th century painter who spent much time in Asia, but he has no proof of this. The buyer has not heard of the famous 19th century painter and is not interested in the picture’s origin. The buyer purchases the painting for £40,000 because he likes it.

Is the art dealer criminally liable for fraud by false representation?

A) He is not liable for fraud by false representation if the art dealer honestly believes that the buyer would have the purchased the painting anyway, knowing its real origins

B) He is liable fraud by false representation as has clearly overcharged the buyer

C) He is not liable for fraud by false representation as his comment about the famous 19th century painter did not influence the buyer in making the purchase

D) He is not be liable for fraud by false representation as the art dealer has not made a false representation

E) He is liable for fraud by false representation

A

E) He is liable for fraud by false representation

(Correct. The art dealer has made a false representation as to his state of mind. This is included in the definition of representation in s 2(3) Fraud Act 2006. He intends to make a gain in terms of money paid for the painting and the jury are likely to consider him to be dishonest on the Ivey v Genting Casinos test.
The other options are incorrect or not the best answer.
While the art dealer is criminally liable for fraud by false representation, overcharging is not the reason why. The cases of overcharging in Silverman and Jones have been applied in circumstances of mutual trust or where the defendant has been a trusted friend; the facts do not suggest this.
The art dealer has made a false representation as to his state of mind, s 2(3) Fraud Act 2006. The art dealer says he believes the artist to be a famous 19th century painter who spent much time in Asia, but he has no proof of this. However, he knows the picture was painted by a friend of the third party he bought the painting from.
It doesn’t matter that the art dealer’s comment about the famous 19th century painter did not influence the buyer in making the purchase. It is not necessary to show a causal link between the representation and what the person making it hopes to achieve.
It doesn’t matter if the art dealer honestly believes that the buyer would have the purchased the painting anyway, knowing its real origins. The negative definitions of dishonesty contained in the Theft Act 1968, section 2(1) do not apply to offences under the Fraud Act 2006. While this might be a relevant consideration if the art dealer was charged with theft, section 2(1)(b) Theft Act 1968 is irrelevant when considering whether the art dealer is criminally liable for fraud by false representation)

182
Q

A woman lost her job and claimed benefits. Later, she acquired well-paid employment. She did not inform the benefits office of her new income, so the benefits continued to be paid into her bank account. She did not spend the continuing benefits as she is saving them for a deposit for a house.

Which of the following best describes the woman’s liability for fraud?

A) The woman will be liable for fraud committed by abusing the position she occupied in which she was expected to safeguard or not act against the financial interests of any other person

B) The woman will be liable for fraud by failure to disclose information which she was under a legal duty to disclose

C) The woman will be liable for fraud by false representation, failure to disclose and abuse of position

D) The woman will not be liable for fraud by failure to disclose until she spends the money paid into her account after she gained her new employment

E) The woman will be liable for fraud by false representation as she has made an implied representation by conduct that she is still unemployed

A

B) The woman will be liable for fraud by failure to disclose information which she was under a legal duty to disclose

(Correct. The facts of this case are very similar to those in R v Mashta. The answer reflects the wording of s 3 Fraud Act 2006 which is committed where a defendant dishonestly fails to disclose to another person information which he/she is under a legal duty to disclose. The woman fails to inform the benefits office of her employment, which she is legally required to do. She is dishonest and intends to make a gain for herself by keeping the money she has received.
The other answers are incorrect.
The actus reus of fraud by failure to disclose is committed as soon as she obtains the new job and fails to inform the benefits office of her employment.
She does not occupy a position in which she is expected to safeguard or not act against the financial interests of any other person for the purposes of fraud by abuse of position.
The case of R v Twaite suggests that pure silence, without an accompanying action, as in Idrees v DPP, cannot amount to a representation for the purposes of fraud by false representation)

183
Q

A man applies for car insurance and fails to put on the form that he made a large claim from the Motor Insurance Bureau. The Motor Insurance Bureau is an organisation which pays out money on behalf of uninsured drivers and the man’s claim arose after an uninsured drunk driver crashed into the man’s parked car.

Which of the following explains the man’s liability for fraud by failing to disclose information he is under a duty to disclose?

A) The man may not be liable as a jury may find him honest if he believes that the previous claim does not make him more likely to make a future claim

B) The man will not be liable as the previous claim does not make him more likely to make a future claim

C) The man will not be liable if he believes that the previous claim does not make him more likely to make a future claim

D) He will not be liable if the insurance company would have kept his premium the same if they knew about the previous claim

E) He will not be liable as the insurance contract is invalid due to his failure to mention the claim, so the insurance company will not suffer any loss

A

A) The man may not be liable as a jury may find him honest if he believes that the previous claim does not make him more likely to make a future claim

(Correct. Although the actus reus aspect of the offence is made out in that he has failed to disclose information which he is under a duty to disclose, the mens rea of the defendant is less clear. The man’s knowledge and beliefs will be considered by the jury when applying the test in Ivey v Genting Casinos and deciding whether he is dishonest by the standards of ordinary decent people and he might be found not to be dishonest. He does have an intent to gain money (a reduced insurance premium) by his omission, which is presumably why he did not put it on the form.
The other options are incorrect or not the best answer.
Just because the man believes that the previous claim does not make him more likely to make a future claim, doesn’t mean that the jury will agree and find him honest.
The defendant must intend to make a gain for himself or another, or to cause loss to another. It is not necessary to show any gain or loss in fact. Whether in fact the previous claim does not make him more likely to make a future claim is irrelevant.
There is no need for a causal link between the failure to disclose and the gain or loss of money or property. Whether the insurance company would have kept his premium the same if they knew about the previous claim is irrelevant. It would also be irrelevant if the insurance contract was invalid due to his failure to mention the claim, for the same reason)

184
Q

A hairdresser runs a small business from a converted garage at her home. She gives her clients that are over 65 years old a 25% reduction. One day she sees an article about one of her clients in a newspaper. She discovers that her 85-year-old client, whose hair she has been doing for 15 years, is extremely rich. The next time that client comes to have her hair done, the hairdresser charges the client the full price, saying she is no longer giving a 25% reduction for those over 65 years old. This is untrue.

Which of the following best explains the hairdresser’s criminal liability for fraud?

A) The hairdresser will not be liable for a fraud offence

B) The hairdresser will be liable for fraud by abuse of position because she has lied, she is therefore dishonest

C) The hairdresser will be liable for fraud by false representation because she has lied, she is therefore dishonest

D) The hairdresser will be liable for fraud by abuse of position because she occupies a position where she would be expected to safeguard or not act against the client’s financial interests

E) The hairdresser will be liable for fraud by false representation if she is found to be dishonest

A

E) The hairdresser will be liable for fraud by false representation if she is found to be dishonest

(Correct. This best explains the hairdresser’s criminal liability for fraud as she has clearly made a false representation with an intention to make a gain in money terms (the increased payment). The false representation is that she is no longer giving a 25% reduction.
Her liability for fraud by false representation will hinge on dishonesty. Just because she has lied does not make her dishonest, see **R v Clarke. **A jury may find her not to be dishonest on the Ivey v Genting Casinos test in these circumstances.
Fraud by abuse of position is unlikely to apply as, although the hairdresser and the client have a long relationship and the client probably trusts her, this relationship concerns nothing of a financial safeguarding nature. It is by no means certain that she occupies the position required for fraud by abuse of position – that she would be expected to safeguard or not act against the client’s financial interests.Whether such a relationship is capable of arising on the facts is a question for the judge to decide on a case by case basis then the jury determine if they are sure that was case. See R v Valujevs and another)

185
Q

A boy really wants to become a scout. However, the scout group in the area where he lives is full, with a long waiting list. The boy’s mother fills in the application form for a scout group in a neighbouring area, giving his address as her sister’s house which is in that area.

Has the mother committed an offence of fraud by false representation?

A) The mother will only have committed fraud by false representation if the boy gets a place in the scout group in the neighbouring area

B) The mother will not have committed fraud by false representation as there is no intent to gain or cause loss by her false representation

C) The mother will not have committed fraud by false representation as she does not intend to make a gain for herself, she intends to make a gain for her son

D) The mother will only have committed fraud by false representation if the scout group she has applied to restricts entrance to children from a specified area and the jury decides she is dishonest

E) The mother will only have committed fraud by false representation if the jury decides she is dishonest.

A

B) The mother will not have committed fraud by false representation as there is no intent to gain or cause loss by her false representation

(Correct. Section 5(2)(a) states that the gain or loss must be in money or other property. The mother does not intend to make such a gain or cause such a loss.
The other answers are incorrect because they do not make this point.
The gain or loss must be in money or other property, whether real or personal.
It is not necessary that the gain be for the person making the false representation. The defendant must intend to make a gain for themselves or another or cause loss to another.
There is no requirement that the mother’s false representation achieves what she wishes to gain i.e. it doesn’t matter whether the scout group restrict entrance to children from a specified area or whether son gets a place in the neighbouring scout group or not. All that is required is that the mother intended to make a gain, even if no such gain arose.
The mother has not committed a fraud in these circumstances, but a possible lack of dishonesty is not the reason why)

186
Q

A man is sitting on a train late at night. There are several other people in his carriage. A drugs dealer who the man has recently swindled in a drugs deal, comes into the carriage and sits on the seat next to the man, blocking him in. The dealer whispers ‘Don’t worry – I’m not going to smash your face in with all these people about.’ The man knows the train is due at its final destination in two minutes and is frightened. The dealer sees this and is pleased.

Which of the following best explains whether the dealer has committed an assault?

A) The dealer has not committed an assault as he did not have the mens rea.

B) The dealer has committed an assault as he has caused the man to apprehend immediate unlawful personal violence

C) The dealer has not committed an assault because words can negate an assault

D) The dealer has not committed an assault as although the man apprehends violence, it is not immediate

E) The dealer has committed an assault as the man is frightened

A

B) The dealer has committed an assault as he has caused the man to apprehend immediate unlawful personal violence

(Correct. Immediate does not mean instantaneous. See Smith v Superintendent of Woking Police Station. It was held by Lord Steyn in R v Ireland that it could be in the next couple of minutes. In the next couple of minutes, the train will reach its final destination and the other people in the carriage will have gone. The man fears being hit at this point which would count as immediate violence. It is very likely that the dealer had intention to cause the man to apprehend immediate violence from the moment he sat down, but certainly he has it when he is pleased the man is frightened and at this point his act of causing the apprehension of immediate unlawful personal violence is still continuing.
The other options are incorrect.
While words can negate an assault, they do not automatically do so. Ultimately, liability will depend on whether the victim did apprehend unlawful personal violence. This scenario can be distinguished from the case of **Tuberville v Savage. **The words ‘Don’t worry – I’m not going to smash your face in with all these people about’ are unlikely to negate an assault in these circumstances because the train is due at its final destination in two minutes.
Evidence that the victim is frightened of the defendant is not sufficient to establish assault. The victim must apprehend immediate and unlawful personal violence.
Liability may be established so long as the defendant has mens rea at some point during the continuation of the actus reus of assault, see Fagan v Metropolitan Police Commissioner. It is very likely that the dealer had the mens rea from the moment he sat down, but he certainly has it when he is pleased the man is frightened and at this point his act of causing the man to apprehend immediate unlawful personal violence is still continuing and intentional)

187
Q

A youth has been kept inside by his mother after she discovered he has been delivering drugs for a gang. The youth’s handler telephones the mother and says, ‘I am outside your house with a brick which I am ready to put straight through your window. Let him out to do his job or you will regret it.’ In fact, the handler is 20 miles away. The mother, fearing that she and her son were about to get hurt, lets her son out.

Which of the following best explains whether the handler has committed an assault?

A) The handler will have committed an assault only if the mother feared she would be hurt by the brick or by broken glass should the handler carry out his threat

B) The handler will not have committed an assault as he has threatened to do criminal damage rather than apply personal violence

C) The handler will not have committed an assault as he gave the woman an option to avoid any harm and she took it

D) The handler will have committed an assault if he saw the risk that the mother would apprehend immediate unlawful personal violence

E) The handler will not have committed an assault as he cannot carry out his threat immediately

A

D) The handler will have committed an assault if he saw the risk that the mother would apprehend immediate unlawful personal violence

(Correct. The mother has apprehended immediate unlawful personal violence. There is no requirement that the handler be able to carry out his threat, as held in Logdon v DPP. The mens rea is intention or recklessness as to causing another to apprehend immediate unlawful personal violence. It is very likely that the handler saw the risk of his words having this effect.
The other options are incorrect.
Where the victim does apprehend immediate and unlawful personal violence, the violence apprehended need not be the same violence as that threatened.
A conditional threat may be the basis of liability for an assault so long as the victim did apprehend immediate and unlawful personal violence)

188
Q

A man spends the evening in the pub and returns home extremely drunk. He takes the lift to his first floor flat and the movement of the lift causes him to vomit all over the lift. This is damage of property belonging to another as effort will have to be expended to return the lift to its original state.

Does the man have the mens rea for basic criminal damage?

A) The man will have the mens rea as a reasonable person would have seen the risk that a person might vomit in a lift when drunk

B) The man does not have the mens rea as he did not intend to vomit in the lift

C) The man does not have the mens rea as voluntary intoxication is a defence to this specific intent crime

D) The man will have the mens rea as voluntary intoxication is no defence to this basic intent crime

E) The man will have the mens rea if when sober, he would have seen the risk of vomiting in the lift while drunk

A

E) The man will have the mens rea if when sober, he would have seen the risk of vomiting in the lift while drunk

(Correct. The man is voluntarily intoxicated, this is a basic intent crime and the jury will be asked this question, as per R v Coley, McGhee and Harris. The answer will depend on whether the man tends to be sick when very drunk and whether it has happened before in similar circumstances.
The other options are incorrect.
The mens rea for basic criminal damage is intention or recklessness as to destroying or damaging to property belonging to another. Mens rea is not confined to intention.
Criminal damage is a basic intent crime not a specific intent crime. An intoxicated defendant will be deemed to be reckless as to causing damage if they would have seen the risk of causing damage when sober.
The test for recklessness is a subjective one (not an objective, reasonable person test). The defendant must see the risk, in this case of causing damage to property belonging to another, and it must be an unreasonable risk to take in the circumstances known to the defendant)

189
Q

A boy balances a bucket of water on the top of the art room door. When the art teacher pushes open the door, the bucket falls. The bucket does not hit the teacher, but she is covered in water.

Which of the following best explains whether the boy committed a battery?

A) The boy has committed a battery indirectly

B) The boy has committed battery directly

C) The boy has not committed a battery as the teacher broke the chain of causation by pushing open the door

D) The boy has not committed a battery as the bucket missed the teacher but has committed an attempted battery

E) The boy has not committed a battery, as he applied force indirectly

A

A) The boy has committed a battery indirectly

(Correct. All the elements of battery are made out. The boy has applied unlawful force to the teacher by an indirect battery. It does not matter that the force felt by the water was not hard. Collins v Wilcockheld that the merest touch counts as force. It seems the boy intended to apply unlawful force to the teacher, at least by the water falling over her.
The other options are incorrect.
Water falling over a victim will be considered the application of force, so it is not an attempted battery.
The teacher did not break the chain of causation by opening the door as her act was not a free, deliberate and informed act, R v Pagett. It is not an informed act as she does not know about the bucket.
A direct battery would be the boy hitting the art teacher or hitting the art teacher with the bucket.
Force can be applied indirectly. This scenario is similar to DPP v K where acid hitting a face from a hand dryer counted as battery)

190
Q

The leading actor in a play has upset the theatre staff by being arrogant, rude and demanding. A stagehand decides to teach the actor a lesson. He slips a drug into the wine glass from which the actor drinks during the play. The drug has a devastating effect. When the actor stands up and walks across the stage, he appears to be drunk. He tries to speak but cannot remember his words. Finally, he is sick all over the stage. The audience laughs. The effect of the drug wears off in the next 30 minutes, but the actor is very upset and feels humiliated. He gives up his part in the play. Despite not being particularly famous, he is reluctant to leave his house as he worries about appearing in public, believing everyone will recognise him and laugh at him again.

Has the stagehand caused the actor grievous bodily harm?

A) The stagehand has not caused the actor grievous bodily harm because serious physical injury is needed

B) The stagehand has caused grievous bodily harm if the actor perceives the harm done to him to be serious

C) The stagehand will have caused the actor grievous bodily harm if what the actor is suffering from is a serious recognised psychiatric illness

D) The stagehand has caused the actor grievous bodily harm as he caused the actor to feel emotions such as upset and worry

E) The stagehand will have caused the actor grievous bodily harm if what the actor is suffering from is a recognised psychiatric illness as all such illnesses are serious

A

C) The stagehand will have caused the actor grievous bodily harm if what the actor is suffering from is a serious recognised psychiatric illness

(Correct. Grievous bodily harm means serious harm, R v Saunders. A recognised psychiatric illness can be actual bodily harm (R v Chan Fook) or grievous bodily harm if serious (R v Ireland).
The other options are incorrect.
A serious psychiatric illness may amount to grievous bodily harm, R v Ireland, but not all psychiatric illnesses will be considered serious.
Mere emotions such as distress, fear or panic do not count as bodily harm, so cannot amount to grievous bodily harm, **R v Chan Fook. **
Whether the harm done constitutes serious harm will be assessed objectively and not merely on the basis of the victim’s perception. Psychiatric injury may amount to GBH if sufficiently serious, but its cause and effect will need to be proved by expert evidence, R v Ireland)

191
Q

A woman picks some wild mushrooms while out on a walk. She brings them home and cooks them. When she tells her husband and son that the mushrooms had been growing wild, the son suggests they might be poisonous and says he is not going to eat them. The woman also decides not to eat any, but her husband says, ‘Don’t be silly. They smell delicious and I’m happy to take the risk, so I’ll eat them.’ The woman puts them on toast and gives them to her husband. He eats them and suffers kidney failure.

Is the woman liable for an offence under s 20 Offences Against the Person Act 1861?

A) She will not be liable as the husband consented

B) She will not be liable as there was no causal link between her acts and the husband’s kidney failure

C) She will be liable as she saw the risk of causing some harm

D) She will be not be liable as she did not intend any harm

E) She will be liable as she saw the risk of causing actual bodily harm or worse

A

B) She will not be liable as there was no causal link between her acts and the husband’s kidney failure

(Correct. To satisfy the mens rea of a s 20 OAPA 1861 offence, the defendant must intend to cause some harm, or see a risk of causing some harm to someone. Although the woman saw the risk of causing her husband some harm which constitutes the mens rea of the offence, the actus reus is incomplete. Kidney failure constitutes serious harm, but it was not caused by the woman. Factual causation is satisfied as but for the woman’s act in serving the man the mushrooms, he would not have suffered grievous bodily harm. However, there is no legal causation because the man’s decision to eat the mushrooms was a free, deliberate and informed act which broke the chain of causation. As in R v Kennedy, he knew there was a possibility that the mushrooms would do him harm and deliberately took the risk.
The husband did consent to take the risk and eat the mushrooms. However, it is not possible to consent to actual bodily harm or worse for no good reason as held by the House of Lords in R v Brown. This rule applies where harm that amounts to actual bodily harm or above, was intended or foreseen by the defendant, as it was here. There are no relevant exceptions that apply here)

192
Q

During a party, the hostess goes into her kitchen where she finds her boyfriend arguing with a man she has never seen before. The man grabs her boyfriend by the shoulders. Thinking the man was a stranger who had not been invited, the woman picks up a frying pan and brings it down on the man’s head, fracturing his skull. In fact, the man is her boyfriend’s cousin who had been invited to the party. The woman believed she was acting to protect her boyfriend.

Which test will be used to determine her whether the woman acted in self-defence?

A) Was the force proportionate?

B) Was the force grossly disproportionate, and if not, was the force reasonable?

C) Was the force reasonable?

D) Was the force reasonable and proportionate?

E) Was the force grossly disproportionate?

A

B) Was the force grossly disproportionate, and if not, was the force reasonable?

(Correct. The answer best reflects the wording of s 76 Criminal Justice and Immigration Act 2008. The woman must honestly believe that it was necessary to use force to protect another (the trigger) – a subjective question. The force used must be reasonable (the response) – an objective question but judged on the facts as the defendant believed them to be.
The woman acted to protect her boyfriend who, she believed, was being attacked by a stranger. When assessing her response, this case should be identified as a householder case (s 76(8A)), as the act took place in a dwelling house, the woman was not a trespasser and she believed the man was a trespasser. First it must be asked if the force was grossly disproportionate, s 76(5A). If it was, the woman cannot use the defence. If it was not, then it must be asked if the force was reasonable, R v Ray (Steven))

193
Q

A new accountant was brought into the man’s office to be introduced. The accountant said ‘hello’ and the man smiled and grasped her hand to shake it. The new accountant pulled her hand away, as she had severe eczema which made contact with her hand painful.

Did the man commit the offence of battery?

A) This was not battery as a handshake cannot be described as force

B) This was a battery as the woman did not consent to having her hand shaken

C) This was not battery as the man was friendly rather than hostile or aggressive

D) This was not a battery as the man did not have the mens rea for the offence

E) This was a battery as a handshake can be described as force

A

D) This was not a battery as the man did not have the mens rea for the offence

(Correct. The actus reus of battery took place when the man grasped her hand as the merest touch constitutes force (Collins v Wilcock), and there is no need for the force to be hostile, rude or aggressive (Faulkner v Talbot). Not all applications of force satisfy the requirements of a battery offence. The use of force must be unlawful, for example without consent, and must be accompanied with the relevant mens rea.
The mens rea is intentional or reckless use of unlawful force on another person. The man intended to and did apply force to the accountant, but not unlawful force. The man would not have realised that the accountant would not have consented to the everyday courtesy of shaking hands- he would have honestly believed the accountant consented. The defence might argue that this is covered by implied consent, the kind of everyday physical contact that is accepted to move around in society (Collins v Wilcock))

194
Q

A woman picks some wild mushrooms while out on a walk. She brings them home and cooks them. When she tells her husband and son that the mushrooms had been growing wild, the son suggests they might be poisonous and says he is not going to eat them. The woman also decides not to eat any, but her husband says, ‘Don’t be silly. They smell delicious and I’m happy to take the risk, so I’ll eat them.’ The woman puts them on toast and gives them to her husband. He eats them and suffers kidney failure.

Is the woman liable for an offence under s 20 Offences Against the Person Act 1861?

A) She will not be liable as there was no causal link between her acts and the husband’s kidney failure

B) She will not be liable as the husband consented

C) She will be not be liable as she did not intend any harm

D) She will be liable as she saw the risk of causing some harm

E) She will be liable as she saw the risk of causing actual bodily harm or worse

A

A) She will not be liable as there was no causal link between her acts and the husband’s kidney failure

(Correct. To satisfy the mens rea of a s 20 OAPA 1861 offence, the defendant must intend to cause some harm, or see a risk of causing some harm to someone. Although the woman saw the risk of causing her husband some harm which constitutes the mens rea of the offence, the actus reus is incomplete. Kidney failure constitutes serious harm, but it was not caused by the woman. Factual causation is satisfied as but for the woman’s act in serving the man the mushrooms, he would not have suffered grievous bodily harm. However, there is no legal causation because the man’s decision to eat the mushrooms was a free, deliberate and informed act which broke the chain of causation. As in R v Kennedy, he knew there was a possibility that the mushrooms would do him harm and deliberately took the risk.
The husband did consent to take the risk and eat the mushrooms. However, it is not possible to consent to actual bodily harm or worse for no good reason as held by the House of Lords in R v Brown. This rule applies where harm that amounts to actual bodily harm or above, was intended or foreseen by the defendant, as it was here. There are no relevant exceptions that apply here)

195
Q

During a party, the hostess goes into her kitchen where she finds her boyfriend arguing with a man she has never seen before. The man grabs her boyfriend by the shoulders. Thinking the man was a stranger who had not been invited, the woman picks up a frying pan and brings it down on the man’s head, fracturing his skull. In fact, the man is her boyfriend’s cousin who had been invited to the party. The woman believed she was acting to protect her boyfriend.

Which test will be used to determine her whether the woman acted in self-defence?

A) Was the force reasonable and proportionate?

B) Was the force proportionate?

C) Was the force grossly disproportionate, and if not, was the force reasonable?

D) Was the force grossly disproportionate?

E) Was the force reasonable?

A

C) Was the force grossly disproportionate, and if not, was the force reasonable?

(Correct. The answer best reflects the wording of s 76 Criminal Justice and Immigration Act 2008. The woman must honestly believe that it was necessary to use force to protect another (the trigger) – a subjective question. The force used must be reasonable (the response) – an objective question but judged on the facts as the defendant believed them to be.
The woman acted to protect her boyfriend who, she believed, was being attacked by a stranger. When assessing her response, this case should be identified as a householder case (s 76(8A)), as the act took place in a dwelling house, the woman was not a trespasser and she believed the man was a trespasser. First it must be asked if the force was grossly disproportionate, s 76(5A). If it was, the woman cannot use the defence. If it was not, then it must be asked if the force was reasonable, R v Ray (Steven))

196
Q

A woman, who never drinks alcohol, goes to a party. One of her friends thinks it would be funny to get her drunk. Her friend gives the woman fruit punch laced with vodka. The woman enjoys the punch, drinks several glasses and becomes very drunk. She tells the friend that she feels odd, and the friend explains that is because the punch is full of vodka. As a result, the woman pours the contents of her glass into the dog’s water bowl. The dog, which is small, drinks the contents of its bowl and becomes ill. The woman has damaged property belonging to another.

Does she have the mens rea for simple criminal damage?

A) She will have the mens rea if she would have seen the risk of damaging the dog had she been sober

B) She does not have the mens rea as she did not intend to endanger the life of the dog

C) She has the mens rea because she is intoxicated

D) She will have the mens rea if she saw the risk of damaging the dog even though drunk

E) She does not have the mens rea because she is intoxicated

A

D) She will have the mens rea if she saw the risk of damaging the dog even though drunk

(Correct. This is involuntary intoxication. This can be used as a ‘defence’ to both specific and basic intent crimes, DPP v Majewski. Simple criminal damage is a basic intent crime. She can use evidence of her intoxication to show she did not form the mens rea. The mens rea of simple criminal damage is intention or recklessness as to damaging property belonging to another. She can use her drunken state to show she did not see a risk of damaging the dog. However, if she did see the risk, she will be liable as a drunken intent is still an intent, R v Kingston.
The other options were incorrect.
· She will have the mens rea if she would have seen the risk of damaging the dog had she been sober- this is the test for basic intent crimes when the defendant is voluntarily intoxicated, R v Coley, McGhee and Harris.
· She does not have the mens rea because she is intoxicated- her involuntary intoxication does not operate as an automatic defence.
· She has the mens rea because she is intoxicated- this is not the test for a basic intent crime, even when the defendant is voluntarily intoxicated, see R v Coley, McGhee and Harris.
She does not have the mens rea as she did not intend to endanger the life of the dog- the mens rea of simple criminal damage is intention or recklessness as to damaging property belonging to another. It would be enough if she saw the risk of damaging the dog)

197
Q

An eighteen-year-old girl is having a party when strangers that have not been invited arrive and go into the back garden where the party is taking place. The girl confronts one of the strangers and tells him to leave or she will call the police. He replies, ‘Not before I have had a kiss’ and puts his arm round the girl, attempting to kiss her. One of the girl’s friends picks up an empty beer bottle and smashes it on the stranger’s head.

Which of the following best describes the question(s) the jury will be asked to consider in relation to the friend’s response for the purposes of self-defence?

A) The jury will be asked to consider if the force was grossly disproportionate

B) The jury will be asked to consider if the force was proportionate

C) The jury will be asked to consider if the force was reasonable, but it does not matter if it was disproportionate, as the exact measure of force cannot be weighed to a nicety

D) The jury will be asked to consider if the force was grossly disproportionate, and if not, was it reasonable?

E) The jury will be asked to consider if the force was reasonable which means proportionate

A

E) The jury will be asked to consider if the force was reasonable which means proportionate

(Correct. The answer reflects the wording of s 76 Criminal Justice and Immigration Act 2008. The friend must honestly believe that it was necessary to use force to protect another (the trigger), a subjective question. The force used must be reasonable (the response), an objective question, but judged on the facts as the defendant believed them to be.
The friend acted to protect the girl who, she honestly believed, was being attacked by the stranger. When assessing her response, this case should be identified as a non-householder case because, even though the stranger is a trespasser, one of the elements in the test for a householder case in s 76(8A) is missing. The force is not used by the friend while in a building, s 76(8A)(b). Therefore, the test is that the force must be reasonable. It will not be reasonable if it is disproportionate, s 76(6). In a non-householder case, to be reasonable, the force used must therefore be proportionate.
The other options are incorrect.
In a non-householder case, it is not appropriate to consider whether the force used by the defendant is grossly disproportionate.
While s 76(7)(a) states that the force cannot be weighed to a nicety, and s 76(7)(b) states that evidence that the defendant only did what they honestly and instinctively thought was necessary constitutes strong evidence that only reasonable action was taken by the defendant, it should also be noted that s 76(6) provides that force cannot be reasonable if it is disproportionate)

198
Q

A man is standing at the top of steps leading onto an aeroplane, arguing with a member of the cabin crew who tells him there is no room for his large hand luggage on board and that it would have to go into the hold. An elderly woman behind him shouts, ‘Come on, don’t be so selfish, we are all waiting to board.’ At that the man turns, puts his face up close to the woman’s and screeches at her, ‘Mind your own business and shut up or I’ll punch you.’ The woman takes a step back in fear and it is only the boy behind, gripping her elbow, who prevents her and others from falling down the steps. She then suffers a heart attack from which she dies. It transpires that she had narrow arteries leading to her heart. The man has been charged with unlawful act manslaughter.

Which of the following best explains whether the man’s act was dangerous?

A) The act will be considered dangerous if a reasonable person saw the risk that the woman might suffer serious harm as a result of the man’s act

B) The act will be considered dangerous due to the thin skull rule

C) The act will be considered dangerous if the man saw the risk that the woman might suffer harm as a result of his act

D) The act was not dangerous as the reasonable person would not have known that the woman had a weak heart

E) The act will be considered dangerous if a reasonable person would have seen the risk of the woman falling down the aircraft steps

A

E) The act will be considered dangerous if a reasonable person would have seen the risk of the woman falling down the aircraft steps

(Correct. This is the test from **R v Church: **‘the unlawful act must be such as all sober and reasonable people would inevitably recognise must subject the other person to, at least, the risk of some harm resulting therefrom, albeit not serious harm.’
The test of whether an act is dangerous is an objective test (from the perspective of the reasonable person) rather than a subjective one (from the perspective of the defendant, here the man).
The act is dangerous if a reasonable person would recognise it must subject the other person to the risk of some harm, not necessarily serious harm.
Although the reasonable person has the same knowledge as the man at the time the unlawful act is committed, R v Dawson, and so would not have known of the woman’s heart condition, it was held in R v JM and SMthat the type of harm foreseen (falling down the aircraft steps) does not have to be the type of harm actually caused (the heart attack).
The thin skull rule relates to whether the man caused the death of the woman which is the fourth aspect of unlawful act manslaughter.
All the other elements of unlawful act manslaughter are satisfied: the man intentionally committed an assault (a crime) on the woman, and this caused her to suffer a heart attack from which she died)

199
Q

A client has been charged with gross negligence manslaughter. He is a safety engineer who failed to complete the necessary maintenance and repairs on construction equipment. This resulted in the death of a builder who was operating one of the faulty pieces of equipment.

Which of the following best explains how the judge will direct the jury on the meaning of gross negligence?

A) Whether, having regard to the risk of death involved, the conduct of the defendant goes beyond compensation between subjects

B) Whether, having regard to the risk of death involved, the conduct of the defendant fell below the standard expected of a reasonable person

C) Whether, having regard to the risk of death involved, the conduct of the defendant was so bad in all the circumstances as to amount to a criminal act or omission

D) Whether the defendant’s conduct was, beyond reasonable doubt, below the standard to be expected of a reasonable person

E) Whether the defendant’s conduct was, on balance, below the standard to be expected of a reasonable person

A

C) Whether, having regard to the risk of death involved, the conduct of the defendant was so bad in all the circumstances as to amount to a criminal act or omission

(Correct. This is how the court in R v Adomako stated the jury should be directed on grossnegligence.The other options are incorrect. ‘On the balance of probabilities’ is the standard of proof which tends to be used in civil rather than criminal cases. ‘Compensation between subjects’ is the object of a civil action in Tort for negligence. The object of a gross negligence manslaughter prosecution is a criminal sanction for the defendant whose negligent act or omission shows such disregard for the life and safety of others that it is exceptionally serious and warrants punishment by the State)

200
Q

A man hosts adventure trips for groups at his chalet. He has made sandwiches which contain some nut paste and sets out the picnic for the group. At lunchtime, the tour guide reminds the man that he left instructions on his voicemail to ensure none of the food contains nuts as the tour guide suffers from a nut allergy. The man did not pick up the voicemail but decides to say nothing as he knows how awkward the tour guide has been. The tour guide dies from an allergic reaction to the nuts.

Which of the following best explains the man’s liability for gross negligence manslaughter?

A) He may be liable as there is a special relationship between the man and the tour guide

B) He may be liable as he created a dangerous situation so will be under a duty of care

C) He is not liable as there is no duty of care

D) He is not liable as this is a failure to act

E) He may be liable as has voluntarily assumed a duty of care

A

B) He may be liable as he created a dangerous situation so will be under a duty of care

(Correct. The basis of liability here is the man’s failure to warn the tour guide of the fact that the sandwiches are made with ingredients containing nuts. Although there is no general duty to act or to prevent harm, R v Smith (William), a failure to act may be the basis of criminal liability if there is a legal duty to act. The most appropriate duty here is based on the fact that the man has breached the duty he owed to the tour guide by creating a dangerous situation (R v Miller), that is by including ingredients containing nuts in the sandwiches, albeit innocently, but then failing to say anything to warn the tour guide when alerted to the allergy. For gross negligence manslaughter, the breach of the duty of care must be the cause the death, carry a serious risk of death and be so serious as to warrant criminal sanction.
The other options are incorrect, or not the best answer.
A voluntary assumption is not the most relevant legal duty to act on these facts. For example, Brett J stated in R v Nicholls that: ‘If a person chooses to undertake the care of a person who is helpless either from infancy, mental illness or other infirmity, he is bound to execute that responsibility and if he by gross negligence allows him to die he is guilty of manslaughter.’
A special relationship it is not the most relevant legal duty to act on these facts. Examples of special relationships are doctors and patients, parents and their children along with spouses)

201
Q

A man was driving his speed boat pulling his friend on water skis. As required for safety, a third person was in the boat to watch the water skier. The boat driver should have been looking forwards. Instead, he was looking back at the water skier, gesturing to him to be more adventurous and cross the wash. Because the driver was not looking forward, the boat hit a swimmer who had come a long way from the shore. The swimmer was killed.

Which of the following best describes the offence the man is potentially criminally liable for?

A) He could be liable for unlawful act manslaughter or gross negligence manslaughter

B) He could be liable for unlawful act manslaughter as it is easier to prove that the act was dangerous than that it was grossly negligent

C) He could be liable for murder as it was virtually certain that the swimmer would suffer serious harm

D) He could be liable for unlawful act manslaughter as the swimmer was killed by an act rather than an omission

E) He could be liable for gross negligence manslaughter

A

E) He could be liable for gross negligence manslaughter

(Correct. It will not be murder as there is no direct or oblique intention to kill or cause serious harm to someone. It was not the driver’s aim or purpose, nor did he appreciate that it was virtually certain that the swimmer would die or suffer serious harm. As the man has caused the death of another but lacks mens rea for murder, he may be liable for involuntary manslaughter. Driving a speed boat is not an unlawful act in itself (although driving the boat dangerously may make it unlawful), so it is not an unlawful act sufficient for unlawful act manslaughter, Andrews v DPP. Gross negligence manslaughter will apply as the man owes a duty of care to the swimmer which has been breached and this causes the death; the breach carries a risk of death and is potentially so serious as to warrant criminal sanction, R v Adomako)

202
Q

The client has been charged with unlawful act manslaughter. She was acting as a security guard at a nightclub. The victim was causing trouble in the club so she ejected him. Shortly afterwards, he returned and rushed at her shouting ‘nobody throws me out of a club’. She pushed him in the chest and he fell over. The victim hit his head on the pavement and sustained injuries from which he died.

Which one of the following is the best advice to give to the client?

A) She should plead guilty to unlawful act manslaughter

B) She should plead not guilty as she has not committed an unlawful act

C) She should plead not guilty as she has not committed an intentional, voluntary act

D) She should plead not guilty as her act was not dangerous

E) She should plead not guilty as she has not caused the death of the victim

A

B) She should plead not guilty as she has not committed an unlawful act

(Correct. She is acting to protect herself from an perceived imminent attack by the victim and, if her action is considered to be reasonable, the defence of self-defence will operate making her act lawful, R v Scarlett.
The other options were incorrect.
Not all of the elements of unlawful act manslaughter are fulfilled, so the client should not plead guilty.
The client has intentionally pushed the victim in the chest which results in his death. She does not need to intend the consequences which flow from the push.
The unlawful act will be dangerous if a sober and reasonable person recognises it carries a risk of causing some harm to the other person, R v Church. Her act of pushing the man in the chest may be considered to be dangerous in that a sober and reasonable person would recognise it carries a risk of causing some harm to the victim.
She has caused the death of the victim in fact and in law. But for her push, the victim would not have fell, hit his head on the pavement and died, R v White. Her push is the substantial (more than de minimis) cause of the victim’s death, R v Hughes and operating cause, as there are no intervening acts to break the chain of causation)

203
Q

A man was the lawful owner of several handguns. The man’s brother was always complaining about his neighbour whose dogs bark all night. The brother frequently said his life was intolerable and one day he would have to do something about the dogs or his neighbour. Last month the brother asked to borrow a handgun, saying, ‘I will use it just once and this will transform my life.’ The brother used it to shoot dead his girlfriend’s husband (the victim) and was convicted of murder.

Which of the following best explains the man’s liability as a secondary party to the murder?

A) The man will not be liable if he did not foresee that his brother might kill or cause serious harm to the victim

B) The man will be liable because he caused the death of the victim, without his gun the murder would not have taken place

C) The man will be liable if he thought his brother might use the gun to either kill the dogs or kill his neighbour

D) The man will not be liable if he did not foresee that his brother might kill the victim

E) The man will be liable if he knew his brother would use the gun for some unlawful purpose

A

C) The man will be liable if he thought his brother might use the gun to either kill the dogs or kill his neighbour

(Correct. It is enough if the man assisted his brother in committing one of a number offences he had in contemplation, DPP for Northern Ireland v Maxwell. If the man envisaged his brother killing someone with the mens rea for murder, it does not matter if he does not know the details such as the victim or the day it would take place, R v Bainbridge.
The mental element in assisting or encouraging a crime is intention to assist or encourage. This requires knowledge of any existing facts necessary for it to be criminal and if the crime requires a particular intent, the man must intend to assist or encourage his brother to act with such intent.
The other options are incorrect.
· The man will be liable because he caused the death of the victim, without his gun the murder would not have taken place- it is not necessary to show there is a causal link between the assistance given by the man and the commission of the offence by his brother. Even if such a link can be established, it is not enough to assist in the murder committed by his brother. To be liable, the man must also have the relevant mens rea.
· The man will be liable if he knew his brother would use the gun for some unlawful purpose- this mens rea is not specific enough.
· The man will not be liable if he did not foresee that his brother might kill or cause serious harm to the victim- if the man intends to give his brother the means to commit a crime but it remains unclear what his brother might do, the man may be liable for the offence committed by his brother if he intends his brother to act with the relevant mens rea)

204
Q

Two boys were throwing large stones at the windows of a disused factory. The victim heard the noise of breaking glass and ran towards the boys yelling at them to stop. Both boys laughed and each picked up another stone and threw it in the direction of the victim. One of these stones hit the victim who suffered a fractured skull. Each boy denied it was their stone which hit the victim.

Can the boys be convicted of s 20 Offences Against the Person Act 1861?

A) Neither boy can be convicted as the joint enterprise was to do criminal damage

B) Both boys can be convicted as they were taking part in a joint enterprise and each one was either the principal or a secondary party

C) Neither boy can be convicted as it cannot be proved which one was the principal offender that threw the stone which hit the victim

D) Both boys can be convicted as they were joint principals taking part in a joint enterprise

E) Neither boy could be convicted as they merely threw stones in the direction of the victim, not at the victim

A

B) Both boys can be convicted as they were taking part in a joint enterprise and each one was either the principal or a secondary party

(Correct. The general rule is if it cannot be proved which of two people committed the crime, both must be acquitted. However, if it can be proved that the one who did not commit the crime as the principal was a secondary party to the crime, then both can be convicted, R v Russell and Russell. Here the boy whose stone did not hit the victim was a secondary party. The boys were on a joint enterprise to damage the property. The s 20 OAPA 1861 offence arose from that joint enterprise and they each would have known that the other would act with the mens rea for s 20 i.e. have seen the risk of causing some harm to a person.
The other options are incorrect.
Where two or more parties are committing a crime together as part of a joint enterprise and one party goes on to commit another crime, the other party is not to be considered automatically as a joint principal for the new offence.
Where two or more parties are committing a crime together (in a joint enterprise), but one party goes on to commit another crime, it is not necessary to show that the other party aided or encouraged the new crime. It is sufficient that the other party was a party to the joint enterprise and had the relevant mens rea for an accessory)

205
Q

A woman goes into a department store intending to take a bottle of perfume and leave without paying for it. The woman reaches for the bottle of perfume, ready to take it but then sees a security guard and pulls her hand back.

Which one of the following best describes the offence she may be criminally liable for?

A) Theft

B) Attempted theft

C) Criminal damage

D) The woman will not be criminally liable for any offence

E) Robbery

A

B) Attempted theft

(This is the correct answer. The woman aims to dishonestly take the perfume belonging to the department store permanently. The only element that is missing is her appropriation (taking or even touching) of the perfume. It doesn’t matter that the department store consents to customers taking perfume bottles off the shelves generally, as appropriation can occur even with the consent of the department store (Gomez). The woman reaching for the perfume is likely to go beyond mere preparation and she will be considered to have embarked on the crime proper, like Tosti (examining a padlock) and **Jones **(in the car).
The other options while plausible were incorrect:
- She could be criminally liable for attempted theft.
- The woman has not yet completed the theft which would take place at the point that she appropriates the perfume bottle.
- As there is no theft here, so there can be no robbery.
- There has been no destruction or damage of property, so there can be no criminal damage)

206
Q

A woman tells her friend that her boyfriend has been having affairs with other women and that, in revenge, she is going to his house that night to scratch the paintwork on his new car. The friend says ‘He deserves it. Go for it!’ The woman makes several prominent scratches on the car.

What is the friend’s liability as a secondary party to this criminal damage?

A) She is not liable as a secondary party as the friend would have committed the criminal damage in any event

B) She has counselled the woman to commit criminal damage and is liable as a secondary party

C) She has abetted the woman to commit criminal damage and is liable as a secondary party

D) She has procured criminal damage and is liable as a secondary party

E) She has aided the woman to commit criminal damage and is liable as a secondary party

A

B) She has counselled the woman to commit criminal damage and is liable as a secondary party

(Correct. Counselling is giving advice or encouragement before the commission of the offence. There has been contact between the parties and a connection between the counselling and the offence, although it does not have to be shown that the friend’s words have had a positive effect on the woman’s conduct, see R vCalhaem.
The other options are incorrect or not the best answer.
She is criminally liable as has counselled the woman- there is no requirement for any causal link between the friend’s words and the woman’s commission of the offence here.
Abetting is incitement or encouragement at the time the offence is committed but this happens before the offence is committed. To procure is to produce by endeavour, that is to bring about the woman’s commission of the offence. The friend cannot be said to have caused the woman to commit the criminal damage here. To aid is to give assistance in the commission of the offence. On the facts the friend has not given the woman any assistance e.g. the location of the car or a method to scratch it)

207
Q

A man was the lawful owner of several handguns. The man’s brother was always complaining about his neighbour whose dogs bark all night. The brother frequently said his life was intolerable and one day he would have to do something about the dogs or his neighbour. Last month the brother asked to borrow a handgun, saying, ‘I will use it just once and this will transform my life.’ The brother used it to shoot dead his girlfriend’s husband (the victim) and was convicted of murder.

Which of the following best explains the man’s liability as a secondary party to the murder?

A) The man will be liable if he knew his brother would use the gun for some unlawful purpose

B) The man will not be liable if he did not foresee that his brother might kill or cause serious harm to the victim

C) The man will be liable if he thought his brother might use the gun to either kill the dogs or kill his neighbour

D) The man will be liable because he caused the death of the victim, without his gun the murder would not have taken place

E) The man will not be liable if he did not foresee that his brother might kill the victim

A

C) The man will be liable if he thought his brother might use the gun to either kill the dogs or kill his neighbour

(Correct. It is enough if the man assisted his brother in committing one of a number offences he had in contemplation, DPP for Northern Ireland v Maxwell. If the man envisaged his brother killing someone with the mens rea for murder, it does not matter if he does not know the details such as the victim or the day it would take place, R v Bainbridge.
The mental element in assisting or encouraging a crime is intention to assist or encourage. This requires knowledge of any existing facts necessary for it to be criminal and if the crime requires a particular intent, the man must intend to assist or encourage his brother to act with such intent.
The other options are incorrect.
· The man will be liable because he caused the death of the victim, without his gun the murder would not have taken place- it is not necessary to show there is a causal link between the assistance given by the man and the commission of the offence by his brother. Even if such a link can be established, it is not enough to assist in the murder committed by his brother. To be liable, the man must also have the relevant mens rea.
· The man will be liable if he knew his brother would use the gun for some unlawful purpose- this mens rea is not specific enough.
· The man will not be liable if he did not foresee that his brother might kill or cause serious harm to the victim- if the man intends to give his brother the means to commit a crime but it remains unclear what his brother might do, the man may be liable for the offence committed by his brother if he intends his brother to act with the relevant mens rea)

208
Q

Two boys were throwing large stones at the windows of a disused factory. The victim heard the noise of breaking glass and ran towards the boys yelling at them to stop. Both boys laughed and each picked up another stone and threw it in the direction of the victim. One of these stones hit the victim who suffered a fractured skull. Each boy denied it was their stone which hit the victim.

Can the boys be convicted of s 20 Offences Against the Person Act 1861?

A) Both boys can be convicted as they were joint principals taking part in a joint enterprise

B) Neither boy can be convicted as the joint enterprise was to do criminal damage

C) Both boys can be convicted as they were taking part in a joint enterprise and each one was either the principal or a secondary party

D) Neither boy can be convicted as it cannot be proved which one was the principal offender that threw the stone which hit the victim

E) Neither boy could be convicted as they merely threw stones in the direction of the victim, not at the victim

A

C) Both boys can be convicted as they were taking part in a joint enterprise and each one was either the principal or a secondary party

(Correct. The general rule is if it cannot be proved which of two people committed the crime, both must be acquitted. However, if it can be proved that the one who did not commit the crime as the principal was a secondary party to the crime, then both can be convicted, R v Russell and Russell. Here the boy whose stone did not hit the victim was a secondary party. The boys were on a joint enterprise to damage the property. The s 20 OAPA 1861 offence arose from that joint enterprise and they each would have known that the other would act with the mens rea for s 20 i.e. have seen the risk of causing some harm to a person.
The other options are incorrect.
Where two or more parties are committing a crime together as part of a joint enterprise and one party goes on to commit another crime, the other party is not to be considered automatically as a joint principal for the new offence.
Where two or more parties are committing a crime together (in a joint enterprise), but one party goes on to commit another crime, it is not necessary to show that the other party aided or encouraged the new crime. It is sufficient that the other party was a party to the joint enterprise and had the relevant mens rea for an accessory)

209
Q

A woman hates her neighbour who is a drug dealer. One day, the woman returns home and sees her neighbour doing repairs to his car at the front of his house. Shortly afterwards, she looks out of a window facing over the gardens at the back of the houses and sees a figure squeezing through a window into the neighbour’s house. She goes to the front of the house and sees her neighbour is about to go into his house. She goes outside and engages him in conversation for 10 minutes to give the intruder more time. The intruder leaves the house with several thousand pounds in cash.

Which of the following best describes the woman’s liability for burglary?

A) She will not be a secondary party as the burglary occurred when the intruder entered the house which was before her act of assistance

B) She will be a secondary party to a s 9(1)(b) burglary as she has aided in the intruder’s commission of the offence

C) She will not be a secondary party to the burglary as the intruder was unaware that they were being aided

D) She will be a secondary party to a s 9(1)(a) burglary as she has aided in the intruder’s commission of the offence

E) She will not be a secondary party to the burglary as there is no causal link between her assistance and the offence

A

B) She will be a secondary party to a s 9(1)(b) burglary as she has aided in the intruder’s commission of the offence

(Correct. To aid is to give assistance, help or support to the principal party in carrying out the offence at any stage before or during the commission of the offence. It is not necessary to show a causal link between the assistance and the commission of the offence or a consensus between the secondary party and the principal. When she gave assistance, the intruder may have already appropriated the cash. However, it was held in R v Hale that appropriation is not an instantaneous act. It was also held that a jury is likely to find the act is still continuing while the intruder is on the premises. So, the appropriation (theft) is still continuing, and the s 9(1)(b) Theft Act 1968 burglary is still continuing. There is no need for the intruder to know of the assistance and there is no need for the assistance to cause the burglary)